Drugs Flashcards

1
Q

What kind of drug is clarithromycin? What is the mechanism of action?

A

Macrolides works via the 50s subunit
N/V common SE and can prolong QT interval, can also cause cholestatic jaundice
Inhibit P450 enzymes in the liver
Measure LFTs if LT treatment needed and measure QT on EG if patient CV unstable
• Avoid with hepatic insufficiency
• Reduce if renal insufficiency
• Avoid in pregnancy but not known harmful
Avoid in cardiac conduction abnormalities or in drugs that can prolong QT

How well did you know this?
1
Not at all
2
3
4
5
Perfectly
2
Q

What do benzothiazepines affect vascularly? And what is an example of a drug

A

Diltiazem
These are calcium channel blockers that affect both the peripheral and the heart vasculature
Often used with Verapamil (Phenylalkylamines) for supraventricular arrythmias

Should not be giving non dihydropyridine calcium blockers (verapamil and diltiazem) with BB w/o close supervision both are negatively inotropic, chronotropic- may give HF, bradykinesia, asystole
Caution with verapamil and diltiazem in patients with poor left ventricular function can worsen heart failure
Avoided in AV nodal conduction delay with non dihydropyridine

How well did you know this?
1
Not at all
2
3
4
5
Perfectly
3
Q

What do Phenylalkylamines affect vascularly? And what is an example of a drug? What side effects can it cause?

A

Verapamil
These are calcium channel blockers which affect mainly the heart
Often used with diltiazem (benzothiazpeines) supraventricular arrythmias

verapamil- can cause constipation, bradykinesia, heart block or cardiac failure (note diltiazem may too due to peripheral effects)

Should not be giving non dihydropyridine calcium blockers (verapamil and diltiazem) with BB w/o close supervision both are negatively inotropic, chronotropic- may give HF, bradykinesia, asystole
Caution with verapamil and diltiazem in patients with poor left ventricular function can worsen heart failure
Avoided in AV nodal conduction delay with non dihydropyridine

How well did you know this?
1
Not at all
2
3
4
5
Perfectly
4
Q

What do Dihydropyridines affect vascularly? What are some examples, uses, and SE?

A

(amlodipine, felodipine, isradipine, lacidipine, lercanidipine, nicardipine, nifedipine, nimodipine, nisoldipine)-
These are principally affect peripheral vascular
Amlodipine and to a less extend nifedipine used for 1st line (black or +55) or 2nd like for hypertension, stroke, MI, death from CVD

Amlodipine and nifedipine: Flushing, headaches, palpitations, peripheral oedema due to vasodilation and compensatory tachycardia

Dihydropyridines avoid in unstable angina as contraction/tachycardia increase O2 demand reflex from vasodilation or in severe aortic stenosis as they may cause collapse

How well did you know this?
1
Not at all
2
3
4
5
Perfectly
5
Q

What kind of drug is indapamide?

A

Thiazide like diuretic

How well did you know this?
1
Not at all
2
3
4
5
Perfectly
6
Q

What drug can cause fibrosis of the lung?

A

amiodarone (others include bleomycin, methotrexate)

How well did you know this?
1
Not at all
2
3
4
5
Perfectly
7
Q

When should you take bisphosphonates, SE and CI

A

Take at least 30 minutes before breakfast with plenty of water + sit-upright for 30 minutes following (this is to avoid oesophageal irritation)
Work by decreasing the osteoclasts
SE
Oesophagitis (when taken orally)-oesophageal ulcers
Hypophosphatemia
Osteonecrosis of jaw- high dose IV therapy- therefore good dental care reducing risk
Atypical femoral fracture especially on long term
CI:
Renally excreted therefore avoid in severe renal impairment
CI in hypocalcaemia
Oral administration is CI in upper GI disorders
Smokers and major dental diseases should exhibit care

As they bind calcium, absorption is therefore reduced with calcium salts (including milk) as well as antacids and iron salts

How well did you know this?
1
Not at all
2
3
4
5
Perfectly
8
Q

What type of infections do aminoglycosides treat? e.g gentamicin, tobramycin, amikacin, neomycin.

What is their MAO AND THEIR MAIN SE

A

Used to treat severe infections especially gram-ve aerobes (e.g pseudomonas aeruginosa)
1. Severe sepsis including when the source is not known
2. Pyelonephritis and complicated UTI
3. Biliary and other intraabdominal sepsis
4. Endocarditis
Topical (e.g neomycin) for:
Bacterial skin, eye or external ear infections

WORK VIA THE 30S subunit- enter via aerobic (therefore do not work against streptococci and anaerobes)

Main SE: nephrotoxicity (potentially reversible - rising creatinine and urea) and ototoxicty (often the later is not noted until resolution of acute infection)

Gentamicin is only given IV monitor plasma [] and adjust to prevent damage

CI: in neonates, elderly and those with renal impairment
Not been given with MGravis due to NM transmission effect

MONITOR RENAL FUNCTION

How well did you know this?
1
Not at all
2
3
4
5
Perfectly
9
Q

What drugs increase the likihood of otoxocity with amino-glycosides?

A

Loop diuretics and vancomycin

How well did you know this?
1
Not at all
2
3
4
5
Perfectly
10
Q

What drugs increase the likihood of nephrotoxocity with amino-glycosides?

A

ciclosporin, platinum chemotherapy, cephalosporins or vancomycin

How well did you know this?
1
Not at all
2
3
4
5
Perfectly
11
Q

What antibiotic is associated with the red main syndrome- erythema, less commonly hypotension and bronchospasm?

A

Vancomycin (gram +Ve- staph aureus)- peptidyloglycan cell walls in gram-ve
Also causes thrombophlebitis
Also causes nephrotoxicity and otoxicity
Increased with amino-glycosides

How well did you know this?
1
Not at all
2
3
4
5
Perfectly
12
Q

Is benzylpenicillin pencillase (B lactamase) sensitive or resistance? and how is it given?

A

Sensitive

Administered by injection (IV or IM) only, as hydrolysis by gastric acid prevents GI absorption. It is prescribed for the treatment of severe infections

How well did you know this?
1
Not at all
2
3
4
5
Perfectly
13
Q

What is co-amoxiclav combined with

A

Amoxicillin plus clavulanic acid (augmentin)

How well did you know this?
1
Not at all
2
3
4
5
Perfectly
14
Q

How does metronidazole work? What major SE does it cause?

A

Enters bacterial cells by passive diffusion. In anaerobic bacteria- reduction of metronidazole generates a nitroso free radical. This binds to DNA- reduce the synthesis and causes widespread damage. DNA degradation and cell death (bactericidal)

AB associated colitis, oral infections or aspiration pneumonia, surgical and gynae infections (Gram-ve anaerobic from colon), protozoal infections e.g trichomonas vaginal, giardias, amoebic dysentery)

NEUROLOGICAL AE: peripheral and optic neuropathy, seizures, encephalopathy
Hypersensitivity and GI upset

Metabolised by P450 inhibitor acetaldehyde dehydrogenase- causes disulriam like reaction- headache, flushing, N/ do not drink for more than 48 hrs during or after treatment

How well did you know this?
1
Not at all
2
3
4
5
Perfectly
15
Q

What kinds of drugs are Tetracycline, oxytetracycline, doxycycline, minocycline, demeclocycline, chlortetracycline, lymecycline

A

Tetracycline drugs- which bind to 30S ribosome- are bacteriostatic
1. Acne vulgaris- esp in inflamed papules, pustules and/or cysts (Propionibacterium acnes)
2. Lower respiratory tract infections e.g infective exacerbation of COPD (e.g haemophilus influenzas), pneumonia and atypical pneumonia (mycoplasma, chlamydia psittaci, Coxiella burnettii or Q fever)
3. Cylamida infection including PID
4. Other infections e.g typhoid, anthrax, malaria, and lyme disease (borrelia burgdorferi)
• Intracellular organisms:
• Chlamydiae (non specific urethritis, psittacosis, trachoma)
• Rickettsiae (e.g Q fever)
• Brucellae (with streptomycin or rifampicin)
• Spirochaetes
Uses
• Tetracyclines used for acne
• Demeclocycline used to treat SIADH
Doxycycline is used vs. prophylaxis against malaria

How well did you know this?
1
Not at all
2
3
4
5
Perfectly
16
Q
What kind of drugs are 
Ciprofloxacin
Moxifloxacin 
Levofloxacin
What is their MOA

What drugs increase their risk of AE

A

Quinolones
Broad spectrum of activity, esp against gram-ve. Bactericidal
Ciprofloxacin- unusual as has significant activity against pseudomonas aeruginosa
Newer quinolones Moxifloxacin and Levofloxacin-enhanced activity against gram +

They inhibit DNA synthesis but reserved for 2nd or 3rd line due to resistance and C.difficle

  1. UTI (mostly gram-ve)
  2. Severe gastroenteritis (e.g due to shigella, campylobacter)
  3. LRT (gram positive and gram negative- therefore moxifloxacin or levofloxacin preferred

SE: GI upset, hypersensitivity, lower seizure threshold, hallucination, inflammation and rupture of muscle tendons, prolong QT and increase arrthymia. Broad spectrum therefore associated with c.diff with cephalosporins

AVOID: increase risk of seizures, children and young adults at risk of arthropathy, RF for QT prolongation, drugs with divalent cations (e.g calcium and antiacids) reduce absorption and efficacy, inhibits certain P450 enzymes, NSAIDS increase seizures, prednisolone increase risk of tendon rupture

How well did you know this?
1
Not at all
2
3
4
5
Perfectly
17
Q

What effect does demeclocycline have in patients with SIADH

A

a tetracycline notable for ability to increase sodium [] in patients with SIADH by blocking binding of ADH to receptor

How well did you know this?
1
Not at all
2
3
4
5
Perfectly
18
Q

What SE and CI does tetracylcines have?

A

• N/V and diarrhoea but they have a lower risk of C.diff than other broad spectrum AB
• Hypersensitivity occurs in 1%
• No cross reactivity with penicillins or other B lactam antibiotics
• Can cause oesophageal irritation they should be taken with water - includes ulceration, irritation, dysphagia
• Photosensitivity (increased sunburn reaction with light)
• Discoloured and/or hypoplasia of tooth enamel
Serious: hepatotoxicity and intra cranial hypertension (headaches and visual disruption)

Avoid in pregnancy, breastfeeding children <12 caution in renal impairment, bind to divalent cations, enhance anticoagulant effect by killing normal bacteria

How well did you know this?
1
Not at all
2
3
4
5
Perfectly
19
Q

How should patients be advised to take tetracylcines

A

swallowed whole with plenty of water when sitting or standing to stop them getting stuck - avoid indigestion and medications with iron or zinc 2hrs before and after taking antibiotic (bind to divalent cations and they cause oesophageal irritation, ulcer, dysphagia)
Should protect skin from sunlight (can cause photosensitivity)

How well did you know this?
1
Not at all
2
3
4
5
Perfectly
20
Q

What drug used in angina can cause tolerance- how do you prevent this

A

TOLERANCE (tachyphylaxis) can occur with prolonged use of nitrates- this can reduce efficacy
• Prevent this, time doses to ensure there is a nitrate free period every day during a time of inactivity, usually overnight
• For example, patients should take x2 a day isosorbide mononitrate morning and mid-afternoon to ensure >12 hrs between pm and am dose
Transdermal patches applied in morning and removed at bed time

How well did you know this?
1
Not at all
2
3
4
5
Perfectly
21
Q

How do nitrates work? What are their common SE?

A

• Nitrates are converted to nitric oxide (NO) increases guanosine monophosphate (cGMP) synthesis and reduces intracellular Ca2+ vascular smooth muscle cells, causing them to relax- results in venous and to lesser extend arterial vasodilation
• Relaxation of the venous capacitance vessels- reduces cardiac preload and left ventricular filling- reduces cardiac work and myocardial oxygen demand, relieving angina and cardiac failure
• Nitrates can relieve coronary vasospasm and dilate collateral vessels, improve coronary perfusion
• Relax systemic arteries, reducing peripheral resistance and afterload
However most of the anti anginal affects are mediated by a reduction of the preload

cause hypotension, lightheadness, flushing, headaches and tolerance

Avoid in hypovolaemia- haemodynamic instability, hypotension
Severe aortic stenosis and CVD such as hypertrophic cardiomyopathy
Avoid with drugs that also lower BP e.g sildenafil or similar

How well did you know this?
1
Not at all
2
3
4
5
Perfectly
22
Q

What is the mechanism of aspirin?

A

Thrombotic events: when platelet rich thrombus forms in atheromatous arteries and occludes circulation

Aspirin: irreversibly inhibits COX- to reduce pro aggregatory factor thromboxane from arachidonic acid- reduces platelet aggregation and risk of arterial occlusion

Occurs at low doses and lasts the lifetime of platelets

How well did you know this?
1
Not at all
2
3
4
5
Perfectly
23
Q

What SE do aspirin cause?

A

GI irritation- peptic ulcer and haemorrhage

Hypersensitivity - bronchospasm

High doses- tinnitus

Life-threatening in overdose: hyperventilation, hearing changes, metabolic acidosis, confusion, followed by convulsions, CV collapse, respiratory arrest

How well did you know this?
1
Not at all
2
3
4
5
Perfectly
24
Q

When is aspirin CI or caution needed?

A

Children <16- Reye syndrome- affects the brain and liver
Third trimester in pregnancy- premature closure of ductus arteriosus
Those with hypersensitivity to aspirin and NSAIDS
Caution: peptic ulcer disease (give gastroprotection, and gout (may trigger an acute attack)

Caution with other anticoagulants and anti platelet drugs

How well did you know this?
1
Not at all
2
3
4
5
Perfectly
25
Q

What CI and SE are ADP receptor anti platelet drugs

A

Bleeding esp after GI, intracranial or surgical procedure
GI upset- dyspepsia, pain, diarrhoea
Can affect platelet #= thrombocytopenia

CI:
Not with active bleeding
Stopped 7 days before elective surgery and other procedures unless risk of stopping > risk of continuing
Caution with renal and hepatic impairment, especially where patients otherwise have increased risk of bleeding

Product- need CYP enzymes to active form to have anti platelet effect
Therefore reduced effect with CYP inhibitors- omeprazole, ciprofloxacin, erythromycin, some antifungals and SSRIS

When need PPI- better to use others lansoprazole or pantoprazole than omeprazole

Prasugrel is also pro drug but less susceptible to interactions

Ticagrelor is not pro drug and interacts with CYP inhibitors- increases risk of toxicity, and induces (which may reduce efficacy)

Co prescription with other anti platelet drugs, anti coagulations e.g heparin, or NSAID increase bleeding risk

How well did you know this?
1
Not at all
2
3
4
5
Perfectly
26
Q

What is Ivabradine, used for, SE, and CI

A

Used as option for third line in angina
In mild to severe chronic heart failure

Reduces HR with minimal impact on BP by acting on the sinus node -heart-rate-lowering agent that acts by selectively and specifically inhibiting the cardiac pacemaker current (If), a mixed sodium-potassium inward current that controls the spontaneous diastolic depolarization in the sinoatrial (SA) node and hence regulates the heart rate
SE
Arrythmia, atrioventricular block, dizziness, headaches, hypertension, vision disorders
CI:
Must be in sinus rhythm - CI if heart rate <70 in angina or 75 in hear failure, heart block or acute MI, or unstable angina, shock, unstable or acute heart failure

How well did you know this?
1
Not at all
2
3
4
5
Perfectly
27
Q

What is ranolazine, used for, SE, and CI

A

Third line in angina

Affect sodium dependent calcium channels- increases coronary flow

Asthenia, constipation, headache, vomiting

Weight <60kg, elderly, moderate- sever congestive heart failure, QT interval prolongation , renal/liver failure

How well did you know this?
1
Not at all
2
3
4
5
Perfectly
28
Q

What is Nicorandil, used for, SE, and CI (remember the anaemic guy in AMU was on this! will help with SE)

A

Third line in angina

K+ channel activator- laterals the vessels also acts like nitrates

Asthenia, dizziness, haemorrhage, nausea, vasodilation, vomiting
But Headache common usually transitory

LVF, hypotension, acute pulmonary odema, hypovolaemia, shock

How well did you know this?
1
Not at all
2
3
4
5
Perfectly
29
Q

What are some examples of low potency steroids

A

hydrocortisone, desonide

How well did you know this?
1
Not at all
2
3
4
5
Perfectly
30
Q

What are some examples of mid potency steroids

A

fluticasone, triamcinolone, fluocinolone.

How well did you know this?
1
Not at all
2
3
4
5
Perfectly
31
Q

What are some examples of high potency steroids

A

mometasone, betamethasone, desoximetasone

How well did you know this?
1
Not at all
2
3
4
5
Perfectly
32
Q

What are some examples of v. high potency steroids

A

clobetasol, ulobetasol, diflorasone

How well did you know this?
1
Not at all
2
3
4
5
Perfectly
33
Q

What are some SE of steroids

A

Atrophy of the skin, hyperpigmentation, straie, telangiectasia, and low response after increased use (tachyphylaxis)

How well did you know this?
1
Not at all
2
3
4
5
Perfectly
34
Q

How does azathioprine work and what do you need to check before? What do you need to check after? what are the SI and CI

A
  1. Maintenance of crohns and UC in remission
  2. Disease modifying in rheumatoid arthritis and autoimmune conditions which are not responding to corticosteroids or other standard treatment
  3. To prevent organ rejection in transplant patient

Prodrug therefore not on it own pharmacologically active but on metabolism converts to substances that are
Main metabolite: 6 mercaptopurine which is further metabolised to active substances
Inhibit synthesis of purines (nucleosides adenine and guanine) therefore inhibit DNA and RBA
Most cells can salvage or recycle purines but lymphocytes are dependent on purine syntheses therefore particularly affected
Metabolism and elimination involves xanthine oxidase and thiopurine methyltransferase (TPMT)- this is reduced in some

MOST SERIOUS: bone marrow suppression - significantly in leukopenia and increased risk of infection (reduced dose or break may needed)
Nausea
Hypersensitivity reaction: diarrhoea, vomiting, rash, fever, myalgia, hypotension and pancreatitis may occur
Others: veno-occlusive disease, hepatotoxicity, lymphomas

TMPT phenotyping should be done before and not prescribed for patients with absent TPMT
Those with reduced levels- should be treated by specialist
Hypersensitivity reactions stop drug
Reduce in hepatic and renal
Not iniatited in pregnancy but may continue if benefits>risks
Risk of infection increased with other immunosuppressants e.g steroids - but may be unavoidable

Not prescribed with xathine oxidase inhibitors such as allopurinol as they reduce its metabolism
Increased risk of leukopenia with myelosuppressive or drugs affecting purine synthesis like trimethoprim
Also may reduce the effect of warfarin

Only really given with specialised
May be high dose for remission, lower for maintenance
Warn to seek urgent advice if: sore throat, fever (infection) bruising, bleeding (low platelet count)or rash, diarrhoea, vomiting, or abdominal pain (hypersensitivity)
TMPT phenotyping should be done before and not prescribed for patients with absent TPMT
Those with reduced levels- should be treated by specialist
Hypersensitivity reactions stop drug
Reduce in hepatic and renal
Not iniatited in pregnancy but may continue if benefits>risks
Risk of infection increased with other immunosuppressants e.g steroids - but may be unavoidable

Not prescribed with xathine oxidase inhibitors such as allopurinol as they reduce its metabolism

Increased risk of leukopenia with myelosuppressive or drugs affecting purine synthesis like trimethoprim
Also may reduce the effect of warfarin

Only really given with specialised
May be high dose for remission, lower for maintenance
Warn to seek urgent advice if: sore throat, fever (infection) bruising, bleeding (low platelet count)or rash, diarrhoea, vomiting, or abdominal pain (hypersensitivity)

Full blood count in the first 4 weeks weekly or after changing dose, and 3 monthly thereafter

How well did you know this?
1
Not at all
2
3
4
5
Perfectly
35
Q

What is Teriparatide

A

Recombinant human parathyroid hormone peptide 1-34 (teriparatide)
Anabolic agent that stimulates bone formation
• Teriparatide reduces vertebral and non-vertebral fractures in post menopausal women with established osteoporosis, although data on hip fracture not available
Recombinant human parathyroid hormone peptide 1-34 (teriparatide)

How well did you know this?
1
Not at all
2
3
4
5
Perfectly
36
Q

What Strontium ranelate

A
  • Dual action bone agent- increases deposition of new bone by osteoblasts and reduces osteoclast
  • Two stable strontium atoms linked to organic acid ranelic acid
  • MOA is uncertain but weak anti resorptive activity while maintaining bone formation
  • Reduces risk of vertebral, hip and other non-vertebral fractures in post menopausal woman
However • Nausea, diarrhea and headaches
• Small increase in VTE 
• CVD concerns
• Recent safety issues therefore not recommended unless any other treatments
Stevens Johnson
How well did you know this?
1
Not at all
2
3
4
5
Perfectly
37
Q

What is Denosumab

A

• Monoclonal antibody to RANKL
• Anti resorptive agent increases bone mineral density and reduces fractures at spine, hip and other non-vertebral sites
• Fracture risk reduction similar to bisphosphonates
However-Due to RANKL having a role in the immune system, denosumab also increases exacerbations of eczema and small increase in severe cellulitis

How well did you know this?
1
Not at all
2
3
4
5
Perfectly
38
Q

What is Raloxifene

A

• Selective oestrogen receptor modulator
• No stimulatory effect in the endometrium but activates oestrogen receptors in the bone
• Prevents BMD loss at spine and hip in post menopausal woman, thought fractures rates only reduced in spine
Also reduces incidence of oestrogen receptor positive breast carcinoma in woman treated for up to 4 years

However, • Prevent bone loss and reduces the risk of veretbral fractures but not yet shown to reduce the risk of non vertebral
• Leg cramps and flushing may occur
• Risk of VTE complications are increased to a degree similar to seen with HRT
• Use associated with a small increase in stroke
Increase menopausal symptoms

How well did you know this?
1
Not at all
2
3
4
5
Perfectly
39
Q

What kind of drug is Metoclopramide, domperidone, what are they mainly used for? What SE does metoclopramide have that domperidone doesn’t

A

Antiemetics Dopamine D2 receptor antagonists
Prophylaxis and treatment of N/V in a wide range of conditions but particularly in the context of reduced gut motility
They act in 2 areas:
- D2 main receptor in the chemotrigger zone
-Act in the gut and promotes relaxation of stomach, lower oesophageal spincter and inhibtis the gastroduodenal []

Metoclopramide acts in the BBB- therefore extrapyramidal SE- ocluogyric crisis- acute dystonic reaction while domperidone does not cross the BBB
This effect is more common in children and young adults

How well did you know this?
1
Not at all
2
3
4
5
Perfectly
40
Q

What kind of drug is Cyclizine, cinnarizine, promethazine ? What are they mainly used for? SE/CI?

A

Anti emetics
Histamine H1 receptor antagonist

Prophylaxis and treatment of N/V in a wide range of conditions but particularly in motion sickness or vertigo

Block the histamine H1 and acetylcholine receptors- most useful for N/V of many conditions but especially vertigo or motion
Drowsiness is the most common SE but cyclizine is least sedating
May give anticholinergic effects: dry throat and mouth, excitation or depression

AVOID: in BPH and other sedating drugs and also hepatic encephalopathy

How well did you know this?
1
Not at all
2
3
4
5
Perfectly
41
Q

What are some DMARBS

A

DMARCS:
Methotrexate, Sulphasalazine, Leflunomide, (Hydroxychloroquine),(Cyclosporine)
Older DMARDsGold,Penicillamine,Azathioprine (Cyclophosphamide)

How well did you know this?
1
Not at all
2
3
4
5
Perfectly
42
Q

What is the MOA of methotrexate, SE, and CI

A

Methotrexate:
Folic acid antagonist
Weekly folic acid supplements are co-prescribed

MONITOR FBC/LFTS
Side effects
- mucosal damage- ulcers
- bone marrow suppression
- liver cirrhosis (LFT)
- pneumonitis -CXR before
-Tetratogenic
-Can increase RA nodule numbers 

Contraindications
- pregnancy- should wait at least 6 months after treatment stop, men should use contraception for at least 6 months after
- renal impairment
- abnormal liver function (as hepatotoxic)
Drug interactions
-NSAIDs,penicillin, folate agonists,
• avoid prescribingtrimethoprimorco-trimoxazoleconcurrently - increases risk of marrow aplasia
• high-doseaspirinincreases the risk of methotrexate toxicity secondary to reduced excretion

How well did you know this?
1
Not at all
2
3
4
5
Perfectly
43
Q

what is the MOA of hydroxychloroquine and SE

A

Hydroxychloroquine
Inhibition of phospholipase A2
Increasing the lysosomal pH in APC
Blocks toll like receptors

LEAST TOXIC DMARD
Side effects
Retinopathy, corneal deposits, myopathy

How well did you know this?
1
Not at all
2
3
4
5
Perfectly
44
Q

What is the MOA of sulsalazine and SE

A

Unknown MOA

Side effects
Oligospermia, rash, bone marrow suppression (leucopenia, thrombocytopenia), nausea and vomiting

How well did you know this?
1
Not at all
2
3
4
5
Perfectly
45
Q

What is the MOA of leflunomide and SE?

A

Pyrimidine synthesis inhibitor- inhibits cell division

Rash, diarrhoea, bone marrow suppression, hepatotoxic and teratogenic

How well did you know this?
1
Not at all
2
3
4
5
Perfectly
46
Q

What kind of drugs are these. what SE and CI do they have?

Etanercept, infliximab, adalimumab, certolizumab pegol, golimumab

A

TNF-a inhibitors
Side effects:
Infections severe to minor, reactivation of TB, Hep B/C , worsening cardiac failure, hepatotoxicity, possible maligancies (skin), haematologic events, neurological events, lupus like symptoms
CI:current/previous neoplasm & infections and congestive heart failure

How well did you know this?
1
Not at all
2
3
4
5
Perfectly
47
Q

What is Abatacept?

A

It is a selective co-stimulator modulator which inhibits T cell activation blockings its interaction with CD80/CD86

How well did you know this?
1
Not at all
2
3
4
5
Perfectly
48
Q

What is sarilumab?

A

works vs. IL-6 R

How well did you know this?
1
Not at all
2
3
4
5
Perfectly
49
Q

What are Tofacitinib and Baricitinib:

A

interferes with the JAK-STAT signaling pathway, which transmits extracellular information into the cell nucleus, influencing DNA transcription.

How well did you know this?
1
Not at all
2
3
4
5
Perfectly
50
Q

What are Rituximab and what SE?

A

B cell depletion (CD20 inhibitor)
Side effects
Severe mucocutaenous reactions, infections, Hep B reactivation

How well did you know this?
1
Not at all
2
3
4
5
Perfectly
51
Q

What drugs do you give in pregnancy for RA

A

Paracetamol is the analgesic of choice

  • Oral NSAID and selective COX2-after implantation up until last trimester if symptoms justify their use
  • Cortosteroidscan be used but main maternal risk- hypertension, glucose intolerance, osteoporosis
  • DMARDS:DO NOT USE METHOTREXATE, LEFLUNOMIDE, CYCOPHOSPHAMIDE, GOLD AND PANCILLAMINE
  • Cytokinemodulators not clear
How well did you know this?
1
Not at all
2
3
4
5
Perfectly
52
Q

What is phytomenadion

A

It is vitamin K given to reverse Warfarin- also can give prothrombin complex

How well did you know this?
1
Not at all
2
3
4
5
Perfectly
53
Q

What is the deal with digoxin?

A
  1. In AF and atrial flutter- used to reduce the ventricular rate. However BB or non-dihydropyridine CCB is usually more effective - however as it relies on PNS tone it tends to be loss in stress or exercise therefore only really used in sedentary patients
  2. Severe heart failure- option when already taking ACI, BB, and either aldosterone antagonist or angiotensin blocker. Used earlier in patients with co-existing AF
Negatively chronotropic (reduces HR)
Positively inotropic (increases force of contraction)

In AF/atrial flutter: indirect pathway involving increased PNS tone, reduces conduction at the AVN preventing impulses being transmitted to ventricles- reducing ventricular rate

In heart failure: direct effect on myocytes --] Na+/K+ ATPase pumps, causing Na+ to accumulate, this causes Ca2+ to accumulate, increasing contractile force  
AE:
Bradycardia 
Gi disturbance
Rash
Dizziness
Visual disturbance 
Range of arrythmias can occur in digoxin toxicity- narrow therapeutic index 
CI:
Worsen conduction problems therefore 
	• Second degree heart block
	• Intermittent complete heart block
	• Ventricular arrythmias or those at risk 
Reduced in renal failure 

Certain electrolyte problems increase risk: hypokalaemia, hypomagnesaemia, hypercalcaemia
Hypokalaemia is the most important as digoxin competes with potassium to bind to the Na+/K+ ATPase pump, when levels are low then effects are reduced
Loop and thiazide cause hypokalaemia increase risk
Amiodarone, CCB, spironolactone, quinin all increase plasma [] therefore increase risk of toxicity

Other:
Oral - 2hrs for effect and IV in 30 mins By either a loading dose is needed.
Check symptoms and HR
Check ECG, electrolytes, renal function

Therapeutic doses can cause ST segment depression- reverse tick sign

How well did you know this?
1
Not at all
2
3
4
5
Perfectly
54
Q

What are some SE of aldosterone antagonists and some CI?

A
  1. Ascites/oedema due to liver cirrhosis-spironolactone 1st line diuretic, increase aldosterone in these conditions
  2. CHF of at least moderate severity or arising 1 month after MI usually with BB or ACE/ARB
  3. Primary hyper aldosterone (conn syndrome ) for patients waiting for surgery or who surgery not appropriate

Aldosterone is produced from the adrenal cortex (zona glomerulosa) mineralocorticoid acts on mineralocorticoid R in distal tubule, increases ENAC activity, increases Na+/water reabsorption, K+ excretion. But antagonist competitively inhibit increased Na+ excretion, increases K+ retention

Hyperkalaemia 
Gynaecomastica 
Impotence 
Liver impairments 
Steven Johnson syndrome

Hyperkalaemia, addison’s (deficient in aldosterone), renal impairment severe, and avoid in pregnancy and breastfeeding

  • Monitor with K+ elevating drugs e.g ACEi and ARBs
    And also supplements

NOTE THAT ONLY EPLERENONE is used for heart failure

Monitor with renal function and serum potassium []

Note that they are week diuretics therefore combinations are used

How well did you know this?
1
Not at all
2
3
4
5
Perfectly
55
Q

What are some SE, CI, and notes about statins (what drugs increase SE)

A
  • Generally safe and well tolerated
  • Headaches , Gi disturbances (nausea, cramps)
  • MUSCLE DAMAGE- myopathy or rarely rhabdomyolysis
  • Rise in liver enzymes
  • Drug induced hepatitis is rare but serious affect

CI:
• Use in caution in hepatic impairment/renal
• Not used in pregnancy- cholesterol is needed for development or breastfeeding
• Decrease metabolism by CYP450 inhibitors e.g amiodarone, diltiazem etc, increases amount in body, therefore SE increases, avoid grapefruit juice
• Myopathy is increased when these drugs are given with fibrates, or ciclosporin avoid unless + >-

Taken orally enters systemic circulation through intestinal cells both passive/active
Major metabolism/elimination through liver vs. kidney
Can cause an increase in liver enzymes (alanine transaminase) therefore check liver tests
Check bloods in 3 months and 12
Alcohol at minimum (rise in ALT up to x3 the upper limit may be acceptable but above this should be stopped)
Check thyroid function- as hypothyroidism may cause hyperlipidaemia and is treatable also increases risk of myositis

Note on for life as stopping increases cholesterol

Taken in evening

Check cholesterol levels for target- for efficacy

Ask patient to report muscle symptoms - weakness or pain

How well did you know this?
1
Not at all
2
3
4
5
Perfectly
56
Q

What is Amiloride

A

K+sparing diuretic work on ENAC in distal convulting part of the kidney (same as aldosterone inhibitors) used to treat hypokalaemia of other diuretics- weak diuretic in own right

How well did you know this?
1
Not at all
2
3
4
5
Perfectly
57
Q

What is Bupropion (Zyban)?

A

Smoking cessation medication
Free on prescription
· Anti depressant- reduces withdrawal cravings
· One of two non- nicotine licensed products
· This is started 1-2 weeks before quite
· Similarly effect to NRT (e.g 9% extra quit)
· Only tested with behavioural support
· CI: epliepsy or increase risk of seizures, eating disorder or bipolar

How well did you know this?
1
Not at all
2
3
4
5
Perfectly
58
Q

What is Varenicline (champix)?

A

· Varenicline tartrate: nicotine receptor partial agonist
· Maintains dopamine levels to counteract the withdrawal
· Start taking 1-2 weeks before you quit
· Reduces the smoking satisfaction
· Similar effect on abstinence as NRT and bupropion i.e 9% extra quit
Caution with psychiatric illness

How well did you know this?
1
Not at all
2
3
4
5
Perfectly
59
Q

Salbutamol. Salmeterol, formoterol, terbutaline- uses, MOA, SE, CI

A

B2 agonists

  1. Asthma- short acting for breathlessness. Long acting as step 3 with inhaled corticosteroids
  2. COPD-short acting and long acting
  3. Hyperkalaemia: nebuliser salbutamol used with insulin, glucose, calcium gluconate for increase K+ into the cells

B adrenergic receptors within the lung SM, dilates and open up airway via Gas- adenyl cyclase to CAMP with ATP, reduce Ca2+ and activate protein kinase A- inactive myosin light chain, also activates the Na+/K+ ATPase- therefore hyperpolarization the cells as K+ moves in- leads to smooth muscle relaxation
However their effects are less reliable than other therapies so not used in isolation

Fight or flight: tachycardia, palpitations, anxiety, tremor
Increase glycogenolysis, increase serum glucose
At high levels lactate may cause muscle cramps- long acting

Care with CVD as tachycardia- angina
Note in asthma- long acting B2 agonists should only be used with ICS

BB may reduce the effectiveness
Concomitate use of high dose nebulised B2 agonists with theophylline and corticosteroids may lead to hypokalaemia

Inhaled however suits patient e.g aerosol or drug powder,

Spacer- may increase airway deposition and treatment efficacy
Short acting- as needed
Long acting- maintenance, x2 regularly may be with steroid in the inhaler

When in hospitalized- nebulized is more often used- note whether to use with air or oxygen (air usually for asthma, and air for COPD due to CO2 retention
- spacer may be used provided not life-threatening
May have combination with steroid

How well did you know this?
1
Not at all
2
3
4
5
Perfectly
60
Q

What are some short acting B2 agonists

A

salbutamol, terbutaline

How well did you know this?
1
Not at all
2
3
4
5
Perfectly
61
Q

What are some long acting B2 agonists

A

salmeterol, formoterol

62
Q

What is the MICA of Benzodiazepines

A

Diazepam, temazepam, lorazepam, chloridiazepoxide, midazolam
USES
1. First line management of seizures and status epilepticus
2. First line management of alcohol withdrawal reactions
3. Sedation for interventional procedures if general anesthesia is unnecessary or undesirable
4. Short term: severe, disabling, distressing anxiety
5. Short term: severe, disabling, distressing insomnia
However for both anxiety and insomnia- non pharmacological treatment (or treatment of the cause is better)

MOA:
• Short acting hypnotic sedative drug with anxiolytic and amnestic properties
• Targets y-aminobutyric acid type A (GABAa) receptor
• This is a chloride channel that opens in response to binding of GABA (inhibitory NT)
• Allows Cl- to move in, making more resistant to depolarization
• Benzos facilitate and enhance binding of GABA to GABAa receptor
• Depresses the synaptic transmission
• Reduces anxiety, increases sleepiness, sedation and anti convulsive effects
• Alcoholic acts on GABA a in chronic excessive use the patient becomes tolerant to presence- abrupt cessation–> excitatory state - therefore introduce a benzo which can be withdrawn in gradual and more controlled
• Reversed by benzodiazepine receptor antagonist
SE:
• Dose dependent
• Drowsiness
• Sedation
• Coma
• Little Cardiorespiratory depression in overdose (compared to opioids)
• But loss of airway reflex can lead to airway obstruction and death
• If used for LT: dependence can develop
• Withdrawal reaction can occur if abrupt cessation occurs
CI:
• Caution with elderly (lower dose)- due to the susceptibility to the effects
• Avoided with high respiratory impairment
• Or with NMD (myasthenia gravis)
• Avoid with liver disease (lorazepam may be best choice due to depending less on the liver ) - this is due to precipitating hepatic encephalopathy
• Additive effect with other sedating drugs e.g alcohol and opioids
• Care with cytochrome p450 inhibitors will increase effect as they depend on cytochrome p450 for elimination- e.g amiodarone, diltiazem, macrolides, fluconazole, protease inhibitors
• Grapefruit juice slows absorption and increases bioavailability

The effects of them are similar what distinguishes them is their duration of actions and dictates what they are used for
• Dose depends on the use whether long acting (usually lorazepam (Initially IVV) or diazepam (IV) seizures, diazepam can also be give rectally)
• IV should only be given where airway management is available
• For alcohol withdrawal oral chlordiazepoxide (long acting) is traditional but diazepam and lorazepam equally acceptable- depends on symptoms and usual alcohol intake
• Sedation a short acting is best e.g midazolam but only used in individuals skilled in safe sedation
• Insomnia and anxiety: immediate acting at lowest effective dose e.g temazepam for shortest period generally no longer than 2 weeks
• Avoid driving and operate complex or heavy machinery
Clinical status, vital signs following IV and high dose oral administration

63
Q

What is Flumazenil

A

antagonist of benzos however use is rarely indicated
Not be given to revere benzo induced sedation when this forms part of mixed or uncertain doses
May worsen seizures which now having been blocked is difficult to treat

64
Q

What kind of drugs are Donepezil, Rivastigmine, Galantamine

A

Anti-Cholinesterase inhibitors:
• Donepezil- OD tablet
• Cheap and similar to prescribe good to tolerate
• Rivastigmine- patch or BD tablets - easier to manage
• Galantamine - this is sedative therefore good if anxious
Use
• For treatment of myasthenia gravis (different drugs though)
• Treatment of AD dementia
• Anesthesia to reverse neuromuscular block due to non depolarizing neuromuscular blockers

Mechanism of action
Inhibit the breakdown of Acetylcholine therefore increase cholinergic neurotransmission both muscarinic and nicotinic
Adverse effects
• Due to the cholinergic actions:
• SM of the bowel is stimulated, sphincter of the bowel and bladder relax– abdominal cramps
• In the eye- cause pupillary constriction, open canal of schlemm
• Nicotinic effects: muscle cramps
• Secretions increase: sweat, saliva, bronchia
• Hallucinations and altered/aggressive behavior
Contraindications
• Avoid systemic treatment in pregnancy and breastfeed, asthma/COPD (increased secretions and bronchospasm), cardiovascular disease (bradycardia and hypotension), peptic ulcer, heart blocks (avoid with NSAIDs, and corticosteroids, and other bradycardia and/or heart block
Note
• Note that organophosphates are irreversible inhibitors
• Ensure that the patient is likely to comply with treatment, as it will not be effective with intermittent administration
• Benefits are modest, ensure they understand this
• Adverse effects can be minimized by starting on low dose, gradually titrate up
• Excessive dose: may lead to cholinergic crisis, characterized by depolarizing muscle block
• Treatment increases risk of convulsions

·

65
Q

When are drugs used in dementia?

A

FOR ALZHEIMER’S DISEASE:
→ Three acetylcholinesterase inhibitors: donepezil, galantamine, rivastigmine as monotherapy
→ Memantine monotherapy- option with moderate AD intolerant or CI to ACHE inhibitors or have severe

NON-ALZHEIMER’S DISEASE:
→ LEWYBODIES: donepezil or rivastigmine mild-moderate & severe, then only galantamine if not tolerate- only memantine if ACHEi are not tolerated or CI
→ VASCULAR DEMENTIA: only consider ACHE inhibitors or memantine if suspected comorbid AD, PD dementia, or dementia with lewy bodies
→ FRONTOTEMPORAL DEMENTIA: DO NOT OFFER ACHEI
→MULTIPLE SCLEROSIS: DO NOT OFFER ACHEI

66
Q

What is Memantine?

A

NMDA receptor antagonist
• Blocks the effects of pathologically raised tonic concentrations of glutamate which may cause neuronal dysfunction
• Used to treat people with moderate to severe AD
• Common AE: dizziness, headache, constipation, somnolence
If cannot tolerate used for behavioral problems in later dementia dampens anxiety

67
Q

What is indicated for insomnia and anxiety- short term

A

Benzo- temazepam- oral time to peak 2-3 hrs, half life 11hrs

68
Q

What are Z drugs?

A

zolpidem and zopiclone
Structurally different from benzo but work similar- enhance GABA mediated neuronal transmission
Developed to overcome some disadvantages of benzo: next day sedation (short HL), dependence and withdrawal
However no evidence of advantage over short acting
Should be used with same caution

69
Q

What is indicated for safe sedation benzo

A

Midazolam- short onset and peak onset
0.5-1 hr peak time, 1-4 HL
Can give parental

70
Q

What benzo is used for withdrawl of alcohol

A

Chlordiazepoxide, and diazepam, given orally and have long half life- active metabolites
But in liver disease: lorazepam

71
Q

Coming off benzos

A

withdrawal very slowly, 10% every 2-4 weeks

With-drawl symptoms- maintain dose until lessen

72
Q

Drugs that cause gingival hyperplasia

A

When your gums grow like this, you usually PANIC

Phenytoin
Acute myeloid leukaemia
Nifedipine (and amlodipine)
Inherited and idiopathic
Ciclosporin
73
Q

What medications increase the risk of VTE?

A

combined oral contraceptive pill: 3rd generation more than 2nd generation
hormone replacement therapy: the risk of VTE is higher in women taking oestrogen + progestogen preparations compared to those taking oestrogen only preparations
raloxifene and tamoxifen
antipsychotics (especially olanzapine) have recently been shown to be a risk factor

74
Q

What are the indications, MOA, SE, CI, and monitoring/nodes of oral corticosteroids (think of the asthma lady)

A

Corticosteroids systemic
ONE

Prednisolone, hydrocortisone, dexamethasone

  1. Allergic or inflammatory disorder (asthma and anaphylaxis)
  2. Autoimmune disorders
  3. Cancers (e.g with chemo) or to reduce tumour swelling
  4. Hormone replacement with adrenal insufficiency or hypopituitary)

Exert mainly glucocorticoid effect (mimic cortisol and anti inflammatory immunosuppress) bind to cytosolic glucocorticoid R to nucleus, increase anti-inflammatory genes and reduce proinflammatory gnus
Directly affect monocytes and eosinophils

Metabolic effect (DM link) increase gluconeogenesis) 
Also have mineralocorticoid - mimic aldosterone and anti inflammatory, immunosuppression- stimulate Na+/H2O retention, k+ excretion in renal tubule 
  • Immunosuppression: infection risk
  • Metabolic: DM and osteoporosis
  • Increase catabolism: muscle weakness, skin thinning, easy bruising, gastritis
  • Mood/behaviour: confusion, suicide, insomnia, psychosis
  • Mineralocorticoid: hypertension, hypokalaemia, oedema (they mimic aldosterone)
    • Supresses pituitary ACTH, decrease stimulation for normal adrenal corticoid may lead to adrenal atrophy, and reduce cortisol
    • May cause an Addison’s crisis with CVD collapse if withdraw quickly (weakness, fatigue, hypotension)

Infection and children - prescribe in caution
Increase peptic ulcer and GI bleed with use of NSAID,
Increase hypokalaemia with B2 agonist, theophylline, loop/thiazide diuretics

Efficacy is reduced by cytochrome P40 induces (phenytoin, carbamazepine, rifampicin)
Reduce immune response to vaccine

Different drugs have different anti inflammatory effects can e given orally, IMIV (dexamethasone highly potent)
Systemic corticosteroid treatments can be given IV, IM, orally
Emergencies (cerebral oedema caused by cancer), dexamethasone is given at high dose then weaned slowly as symptoms improve)
Acute asthma: prednisolone given orally daily
Where oral administration is not appropriate e.g IBD flares, anaphylaxis’s, IV hydrocortisone may be used
In long term treatment e.g for inflammatory arthritis use lowest dose of oral prednisolone that controls disease while limiting adverse effects may require co prescription of steroid spearing agents e.g azathioprine, methotrexate). Also consider the use of bisphosphonates and PPIs to reduce some steroid side effects

DO NOT STOP THEM SSUDDENTLY

Give them steroid car to carry with them always and show if need treatment

Monitor efficacy and monitor adverse effects (e.g glucose HbA1c, and DEXA scan)

Note: that patients on LT corticosteroid therapy have atrophic adrenal glands therefore may be unable to increase cortisol secretion in response to stress, may therefore need to provide this increase cortisol secretion in response to stress, may therefore need to provide artificially by increasing the dose of exogenous corticosteroid

X2 this dose during acute illness, then back to maintenance dose

TAKE IN AM- to mimic natural circadian rhythm and reduce insomnia
Should feel better w/I 1-2 days but do not stop suddently

75
Q

What are some common indications and MOA of beta blockers?
Atenolol, metoprolol, bisoprolol (B1 selective)
Propranolol (non selective beta blockers)

A
  1. Used first line in ischaemia heart disease to improve symptoms and prognosis associated with angina and ACS
    Prophylaxis vs. angina
    Adjacent to treatment of or after MI
  2. Chronic heart failure as first line to improve prognosis
  3. AF as first line to reduce ventricular rate and in paroxysmal AF to maintain sinus rhythm
  4. SVT as first line win patients without circulatory compromise to restore sinus rhythm
  5. Hypertension although not indicated for initial therapy, may be used when others are insufficient/ inappropriate
  6. Symptomatic relief in hyperthyroidism, severe/anxiety/panic
  7. Prophylaxis against migraine headache
  8. Treatment of glaucoma (topically)
  9. Treatment of benign essential tremor
  10. Non selective are used in cirrhosis and to prevent and treat variceal bleeds

Act via competitive antagonism- reversible

B1 receptors are on the heart, while B2 are on the SM of airways/blood vessels.

Via the B1 receptors, they decrease the contraction, speed of conduction, decrease the cardiac work and O2 demand and improve HF via protective measures against the effects of chronic sympathetic stimulation. They slow AF by prolonging the refractory period at the AVN. SVT often involves a re-entry circuit that take sin the AV node, Bb break this and restore sinus rhythm. Bb lockers decrease hypertension via renin response via b1 recpetors.

B1 antagonist: decreased the force and rate of contraction

B2 antagonist (not clinically useful as block bronchodilation): constriction mainly in skeletal muscle, constriction of bronchial muscle, contraction of the GI, inhibits glycogenolysis, gluconeogenesis, lipolysis, contraction of the uterus, and contraction of the detrusor muscle

76
Q

What are some common SE and CI of BB

A

Common side effects: fatigue, cold extremities (Raynaud’s phenomenon), headache, GI disruption, nausea, sleep disruption, nightmares, impotence in males - ERECTILE DYSFUNCTION

Propranolol is dangerous in asthma but usually ok in COPD, can potentially use a selective B1 (atenolol, metoprolol, bisoprolol rather than non selective but generally avoid

In HF usually start low dose then increase (not in acute or unstable)

Avoid in haemodynamic instability

Not in heart block

Note start at a low dose and increase in hepatic failure

Not in pregnancy unless needed (Intrauterine growth retardation, fetal bradycardia, hypoglycaemia)

Not to be given with dihydropyridine Ca2+ channel blockers (verapamil, diltiazem) may cause HF, bradycardia, asystole

NOTE: they can mask physiological response to hypoglycaemia

77
Q

What are macrolides used for what is their MOA

A

Macrolides

Both bactericidal and bacteriostatic
Similar antibacterial spectrum to those of simple penicillin’s e.g benzylpenicillins

Erythromycin, azithromycin, clarithromycin, telithromycin (a ketaloid), spiramycin (specialised use for treatment of toxoplasmosis during pregnancy

Related drugs
Clindamycin, commonly cause AB associated diarrhoea- specialised use for bone infection used in combination w/ other AB

Gram +ve cover

  • Similar antibacterial spectrum to simple penicillin’s, therefore commonly used if patient allergic to penicillin’s
  • Do not penetrate the CNS so cannot be used for meningitis
  • Also active against atypical pneumonias bacteria (mycoplasma, chlamydia, legionella)
  • Azithromycin has slightly different spectrum from erythromycin:
    • Covers Haemophilus influenzae
    • Can be given as single dose for treatment of chlamydial and non gonococcal urethritis)
  • Erythromycin is used topically for treatment of acne
  • Erythromycin stimulates motilin receptors in the gut- increases motility and is used therapeutically on ICU and in patients with diabetic autonomic neuropathy

• Works via the 50S subunit of the bacterial ribosome exact mechanism is unclear but inhibits the bacterial protein synthesis
Erythromycin can increase motilin receptors- stimulation of gut motility but can result in nausea and vomiting

78
Q

What drugs are Cefalexin, cefotaxime, meropenem, ertapenem, Cefuroxime?

A

Cephalosporins and carbapenems

note that cefotaxime is used 2nd line if penicillin mild allergy in sepsis

• Oral cephalosporins are 2nd and 3rd line for urinary and respiratory tract infections
IV are for treatment of infections that are very severe, complicated, or caused by antibiotic resistant organisms

Inhibit peptidoglycan cross linking
Inhibit bacterial cell wall synthesis - causing bacterial cell swelling and death

Cephalosporins: have increasing generations (1st-5th) increased activity vs. gram- and less oral

More resistant to beta lactamases than penicillin

79
Q

SE of cephalosporins and carbapenemes

A
  • Gi upset- nausea and diarrhoea
  • Antibiotic associated colitis - can be complicated by colonic perforation and death
  • Hypersensitivity
  • Increase risk of CNS toxicity including seizures when carbapenems prescribed in high dose or to patients with renal impairment
  • Candida in oropharynx
  • Coombs positive haemolytic anaemia
  • Bone marrow suppression (thrombocytopenia)
  • Inferencing with normal clotting- haemorrhage
  • Caution in risk of C. difficle infection especially in hospital and elderly
  • Allergy to penicillin, cephalosporin or carbapenem especially in anaphylaxis
  • Caution with epilepsy in carbapenems
  • Dose reduction in renal impairment
  • Broad spectrum: enhance anticoagulation of warfarin y killing normal gut flora that make vitamin K
  • May increase nephrotoxicity of aminoglycosides
  • Reduce plasma []and efficacy of valproate

NOT TO BE USED WITH PENCILLIN ALLERGIES

80
Q

Anti platelet drugs ADP receptor antagonists

Clopidogrel, ticagrelor, prasugrel- MOA, SE, CI and monitoring (particularily PPIs and which are pro drugs etc)

A

Prevent platelet aggregation and reduce the risk of arterial occlusion by binding irreversibly to adenosine diphosphate (ADP) receptors (O2Y12 subtype) on surface of platelets. Synergistic actions to COX pathway with aspirin

Bleeding esp after GI, intracranial or surgical procedure
GI upset- dyspepsia, pain, diarrhoea
Can affect platelet #= thrombocytopenia

Not with active bleeding
Stopped 7 days before elective surgery and other procedures unless risk of stopping > risk of continuing
Caution with renal and hepatic impairment, especially where patients otherwise have increased risk of bleeding

Clopidrogrel-Pro drugs- need CYP enzymes to active form to have anti platelet effect
Therefore reduced effect with CYP inhibitors- omeprazole, ciprofloxacin, erythromycin, some antifungals and SSRIS

When need PPI- better to use others e.g lansoprazole or pantoprazole

Prasugrel is also pro drug but less susceptible to interactions

Ticagrelor is not pro drug but interacts with CYP inhibitors- increases risk of toxicity, and inducers which may reduce efficacy

Co prescription with other anti platelet drugs, anti coagulations e.g heparin, or NSAID increase bleeding risk

Clopidogrel only given in oral Low doses- need a week to get fully effect, when rapid is needed, give a loading dose in once only then a regular. When given for drug eluting stent- continue for 12 months! - surgery should be delayed in this time.

UNLIKELIKE ASPIRIN CAN BE GIVEN WITH OR OUTOUT FOOD

In emergency surgery- may need platelet infusion to stop bleeding may be needed

Clopidogrel acts irreversibly therefore the lifespan of the platelet (7-10 days) for the effect to wear off

81
Q

Of beta blockers which are B1 selective which are non selective
Which do you uses in heartfailure

A

Atenolol, metoprolol, bisoprolol,nebivolol (B1 selective)
Propranolol, Carvedilol (non selective beta blockers)

Carvediol, bisoprolol, Metoprolol succinate or nebivolol)

82
Q

What is ranitidine what the advantage/disadvantage of this drugs over others of the same indication

A

H2 receptor antagonist (Ranitidine- also called Zantac)

  • Peptic ulcer disease for treatment and prevention of gastric and duodenal ulcers and NSAID ulcers- more effective is PPI
  • GORD and dyspepsia for release of symptoms (again PPI are more effective in severe cases)
  • H2 blockers reduce gastric acid secretion
  • Acid produced in the proton pump of parietal cells, secrets H+ into lumen, in exchange for K+ in
  • Regulated by histamine
  • Histamine is released by local paracrine cells and binds to H2 receptors on gastric parietal cells
  • Via secondary messenger- this activates the proton pump
  • H2 blockers cannot completely suppress the gastric acid production as they are stimulated by other means

HOWEVER: One advantage of H2, is that they have the rapid onset of effect. Better choice for suppress of acid secretion preoperatively (with general anaesthesia the acid can be refluxed then aspirated- therefore prescribe H2 antagonist)

  • Few side effects
  • Bowel disturbances (diarrhoea and less commonly constipation), headache, and dizziness
  • Renal impairment lower doses
  • No major drug interactions
  • Note alarm symptoms
  • May disguise symptoms of gastric cancer (note alarm symptoms before and during treatment)
  • May have repeat endoscopy to see if healing with ulcer
  • Patients symptoms are best for dyspepsia and GORD
  • Excreted by kidney (lower doses with renal impairment)
  • Can be brought over the counter but only for short term <2 weeks
  • Dose varies with indication
83
Q

Anti psychotics first generation (Typical)

haloperidol, chlorpromazine, prochlorperazine
What are the indications, MOA

A
  1. Urgent treatment of severepsychomotor agitationthat is causing dangerous or violent behaviour, or to calm patients to permit assessment.
  2. Schizophrenia,particularly when the metabolic side effects ofsecond-generation (atypical) antipsychoticsare likely to be problematic
  3. Bipolar disorder,particularly in acute episodes of mania or hypomania
  4. Nausea and vomiting,particularly in the palliative care setting.

Block post synaptic dopamine D2 receptors
Three main dopaminergic pathways within the brain: mesolimbic and mesocortical: run between the midbrain and the limbic and frontal cortex
D2 blockage here is the main determinant of anti psychotic
Nigrostrial pathway connects the substantial nigra with the corpus striatum of BG
Tuberohypophyseal pathway connects the hypothalamus with the pituitary gland
Therefore the blockage of these pathways explain some of the AE
D2 receptors are also found in the chemoreceptive trigger zone where blockage is for their use of nausea and vomiting
All antipsychotics but particularly chlorpromazine have some sedative effect- may be beneficial in acute psychomotor agitation

84
Q

Anti psychotics first generation (Typical)

haloperidol, chlorpromazine, prochlorperazine
What are the SE and CI

A

Extrapyramidal effects: movement abnormalities due to nigrostriatal pathway blockage- is the main SE:
• Acute dystonic reactions: involuntary parkinsonian movements or muscle spasm
• Akathisia: inner restlessness
• Neuroleptic malignant syndrome: rare but LIFETREATNING: rigidity, confusion, autonomic reregulation and prexia
• ALL OCCUR EARLY ON
• LATER: tardive dyskinesia is pointless, involuntary, repetitive e.g lip smacking)- disabling and does not often resolve- this is seen after months and years
Drowsiness
Hypotension
QT prolongation (arrythmias)
Erectile dysfunction
Hyperprolactinaemia (tuberohypophyseal D2 blockage): menstrual disturbance, galactorrhoea, breast pain

Elderly are sensitive- therefore start lower
Avoid in dementia- increase risk of death and stroke
Avoid in PD due to extrapyramidal
Consult BNF when prescribing as extensive interactions: especially drugs that prolong QT interval e.g amiodarone, macrolides

For regular treatment for schizophrenia- started and adjusted under psychiatric
Single dose to control acute or violent behaviour
Haloperidol first line
Dosage depends on guidance
Need different doses
Nausea: haloperidol in regular oral or SC doses or component of continuous SC infusion

For regular: orally or slow release IM depot

Emergencies: Im and occasionally IV for rapid control
IV only when clinicians management neurological and cardiac SE: torsade de points (VT)

ADHERENCE is huge issue- good communication of aims and SE

Antipsychotic effects may take several weeks and dose may need to be adjusted

Dose response relationship is unpredictable when high dose use- therefore monitor

—Chlorpromazine and haloperidol are also licensed for the treatment of intractable hiccups, as is metoclopramide. This can be a very distressing condition which is difficult to treat. A variety of non-pharmacological manoeuvres (e.g. Valsalva manoeuvre, breath holds, sipping ice-cold water) may be tried first. In our experience, they rarely work in intractable cases. However, we have found chlorpromazine (e.g. 25 mg orally) to be effective.

85
Q

What is Neuroleptic malignant syndrome?

A

rare but LIFETREATNING: rigidity, confusion, autonomic reregulation and prexia
life-threatening reaction that can occur in response to neuroleptic or antipsychotic medication typical >atypical
DUE TO DOPAMINE ANTAGONISTS- antipyschotics, dopaminergic drugs (e.g levodopa) for PD- when stopped or reduced. It occurs within hours to days of starting an antipsychotic (antipsychotics are also known as neuroleptics, hence the name)

The pathophysiology is unknown but one theory is that the dopamine blockade induced by antipsychotics triggers massive glutamate release and subsequent neurotoxicity and muscle damage.

Neuroleptic malignant syndrome is characterised by;
• High temperature and fever
• Excessive sweating
• Muscle stiffness
• Feeling mentally confused and disoriented
• Feeling delirious – you may see and hear things that are not there
• Drooling or dribbling saliva
• Unstable blood pressure
• Faster breathing, racing pulse and irregular heart beat
• Incontinence, wetting yourself
• Abnormal blood tests
• Muscle breakdown
• Fits (seizures)
• In serious cases, losing consciousness, going into coma and death

A raised creatine kinase is present in most cases. Acute kidney injury (secondary to rhabdomyolysis) may develop in severe cases. A leukocytosis may also be seen

Management
stop antipsychotic
patients should be transferred to a medical ward if they are on a psychiatric ward and often they are nursed in intensive care units
IV fluids to prevent renal failure
dantrolene may be useful in selected cases
thought to work by decreasing excitation-contraction coupling in skeletal muscle by binding to the ryanodine receptor, and decreasing the release of calcium from the sarcoplasmic reticulum
bromocriptine, dopamine agonist, may also be used

86
Q

What is serotonin syndrome

A

TRIAD: autonomic hyperactivity, altered mental status, NM excitation- usually responds to treatment withdrawal and supportive therapy
Symptoms include hyperreflexia, myoclonus, ocular clonus
Less lab findings
DUE TO SEROTONERGIC AGENTS
Rare when only one serotonergic drug is used, but risk is increased when two or more are used.
· Serotonin precursors (tryptophan)
· Serotonin agonists (triptans, buspirone, LSD)
· Serotonin releasers (MDMA, ecstasy, amphetamines, sibutramine)
· MAOIs
Others: (chlorphenamine, pethidine, cocaine, tramadol, levodopa, bromocriptine, lithium, st johns wort)

Symptoms are often seen within 24hrs of starting/changing therapy and usually resolves within a few days

87
Q

What is mirtazapine?

A

ANTI DEPRESSANT Noradrenergic and specific serotonin antidepressant (NASSA)

  • Treatment of depression where first line of SSRI are ineffective or not tolerated
  • Generalized anxiety disorder and social anxiety disorder (LESS COMMONLY THAN VENLAFAXINE)

Antagonist of inhibitory presynaptic A2 adrenoceptors-increases availability of monoamines for neurotransmission, which appears to be mechanism for where they improve mood and physical symptoms in moderate to severe but not mild depression

Works as a antagonist of histamine but not muscarinics

  • GI upset (e.g drug mouth, nausea, change in weight, diarrhea or constipation)
  • CNS changes (headache, abnormal dreams, insomnia, confusion, convulsions)
  • Causes sedation
  • Less common: hyponatremia and serotonin syndrome
  • Suicidal thoughts and behavior may increase
  • Sudden drug withdrawal: GI upset neurological and influenza like symptoms and sleep disturbance
  • Elderly are at particular risk
  • Dose reduction with renal or hepatic failure
  • Combinations with other antidepressant drugs

Sedative effects may be less severe at higher doses, as the antihistamine effects are higher when low doses, augmented monoamine transmission at high doses counteracts the sedative
However as it is not proven do not advocate increasing dose to overcome sedation or somnolence

Should be taken at night to minimize or benefit the sedative effects

  • Review at 1-2 weeks after starting and then regularly after
  • No effect seen at 4 weeks consider changing the dose or drug
  • Otherwise do not adjust until 6-8 weeks of therapy
  • If want to withdraw- then do so slowly as stopping sudden can cause flu like withdrawal symptoms
88
Q

What is Venlafaxine?

A

ANTI DEPRESSANTS Serotonin and noradrenaline reuptake inhibitors (SNRI)

  • Treatment of depression where first line of SSRI are ineffective or not tolerated
  • Often used as adjunctive in severe or resistance
  • Generalized anxiety disorder and social anxiety disorder

Serotonin and noradrenaline reuptake inhibitor (SNRI) interfering with uptake of these neurotransmitters from the synaptic cleft, increases availability of monoamines for neurotransmission, which appears to be mechanism for where they improve mood and physical symptoms in moderate to severe but not mild depression

Much weaker antagonist of muscarinic and histamine receptors than tricyclic

  • GI upset (e.g drug mouth, nausea, change in weight, diarrhea or constipation)
  • CNS changes (headache, abnormal dreams, insomnia, confusion, convulsions)
  • Less common: hyponatremia and serotonin syndrome
  • Suicidal thoughts and behavior may increase
  • Prolongs QT interval- can increase ventricular arrhythmias
  • Sudden drug withdrawal: GI upset neurological and influenza like symptoms and sleep disturbance- greater risk with this drug compared to mirtazapine
  • Elderly are at particular risk
  • Dose reduction with renal or hepatic failure
  • Cautioned use if at all with CVD associated with increased risk of arrhythmias
  • Combinations with other antidepressant drugs can increase risk of adverse effects such as serotonin syndrome
  • Review at 1-2 weeks after starting and then regularly after
  • No effect seen at 4 weeks consider changing the dose or drug
  • Otherwise do not adjust until 6-8 weeks of therapy
  • If want to withdraw- then do so slowly as stopping sudden can cause flu like withdrawal symptoms
89
Q

What is the tyramine cheese reaction

A

severe hypertension, headache, palpitation, sweating and nausea and vomiting- due to ingesting tyramine rich foods or coadministration with other drugs that potentiate aminergic neurotransmission

90
Q

What are Monoamine oxidase inhibitors (MOAS)
Non selective MAOIS:
Phenelzine, isocarboxazid, tranylcypromine, iproniazid

Inhibitor of MAO: moclobemide (Reversible)

Inhibitor of MOAb:
Selegiline and rasagiline

A
  • Second line in depression (non selective inhibitors and moclobemide) especially if there are phobic or other atypical symptoms especially hysteria
  • Treatment of PD (selegiline and rasagiline)

Amine neurotransmitters e.g 5HT, catecholamines, dopamine) get converted via monoamine oxidase (MOA) to inactive metabolites
MOA has two isoforms:
MAOA: principally in the liver and gut
MAOB: principally in the brain

Monoamine oxidase inhibitors (MOAI) inhibit the breakdown increasing their synaptic levels.

  • Very common effects
  • Autonomic: dry mouth, blurred vision, postural hypotension, blurred vision
  • GI adverse effects: nausea and vomiting
  • They cause hyponatremia: confusion and seizures
  • They can cause tyramine cheese reaction- severe hypertension, headache, palpitation, sweating and nausea and vomiting- due to ingesting tyramine rich foods or coadministration with other drugs that potentiate aminergic neurotransmission
  • Avoid in people who have Cerebrovascular disease who are more susceptible to adverse effects
  • pheochromocytoma
  • Avoid them in cardiac disease, hypertension, arrythmia
  • CNS stimulation therefore avoid with history of psychotic illness
  • Can cause idiosyncratic liver damage avoid in hepatic insufficiency
  • Not with tyramine rich foods or coadministration of drugs increasing aminergic neurotransmission
  • With drugs that increase serotonin syndrome
  • Can potentiate oral hypoglycemic drugs
  • Little evidence in pregnancy but use when the benefits are compelling
  • Although little used today, sometimes used in combination with other antidepressants this increase the incidence of serious adverse effects wait 14 days
Other amine drugs: e.g ephedrine, pseudoephedrine (common components of decongestant cough an cold cures) 
Dopamine receptor agonists 
Direct sympathomimetics 
Indoramin (alpha adrenoceptor antagonist
Tyramine rich foods:
Cheese (especially cheddar) but not cottage or cream
Meat and yeast extracts
Some red wines 
Hung game and poultry 
Pickled herring 
Broad beans
Alcoholic and DE alcoholic beverages
91
Q
What are tricylic antidepressants (think tri= most SE- muscarinic, histamine, adrenergic, dopamine)
Sedative: 
	• Amitriptyline 
	• Clomipramine 
	• Dosulepin (dothiepin) 
	• Doxepin *
	• Maprotiline 
	• Mianserin *
	• Trazodone *
	• Trimipramine
Less sedative:
	• Amoxapine 
	• Imipramine 
	• Lofepramine*
	• Nortriptyline 
*Drugs with fewer anti cholinergic effects
•
A
  • As second line treatment for moderate to severe depression (Where SSRIs are not effective)
  • Treatment for panic disorder
  • Treatment for nocturnal enuresis (unlicensed)
  • Treatment of painful diabetic neuropathy (unlicensed)
  • Prophylaxis vs. migraine (unlicensed)

Inhibit neuronal reuptake of serotonin and noradrenaline from synaptic clef therefore increase availability for neurotransmission. This appears to be mechanism by which they improve the mood, and physical symptoms in moderate- to severe (but not mild) and probably accounts for their effect in modifying neuropathic pain.
Tricyclic antidepressants block a wide array of receptors including muscarinic, histamine (H1), a adrenergic (A1 and A2) and dopamine (D2 receptors) therefore there are a wide arrange of clinical adverse effects that limit their usefulness

Central and peripheral anticholinergic effects they are responsible for the various adverse effects caused by tricyclic antidepressants
• Blocking of anti muscarinic receptors- dry mouth, constipation, urinary retention, blurred vision
• Some block H1 histamine receptors- therefore leading to drowsiness
• Blocking the a1 receptors lead to hypotension (20%)- tolerance however develops therefore try to persevere
• Cardiac adverse effects (multiple mechanisms) include arrythmias, and ECG changes (including prolonging QT and QRS durations)
• In brain more serious effects, convulsions, hallucinations and mania
• Cause syndrome of inappropriate ADH secretion- hyponatremia with confusion(esepcially in elderly)
• Blocking the dopamine receptors: breast changes, sexual dysfunction, and rarely extrapyramidal symptoms of tremor and dyskinesia
• Overdose: severe hypotension, arrhythmias, convulsions, coma, respiratory failure
○ Anticholinergic: hot dry skin, dry mouth, tongue, dilate d pupils, urinary retention
○ Cardiac: sinus tachycardia, prolonged PR and QT
○ CNS: drowsiness, rapid progression to comma, twitching and myoclonic jerks or generalized seizures
• Sudden withdrawal: GI upset, neurological and influenzas like symptoms and sleep disturbance

• Should be used in caution in people who are at high risk of adverse effects: elderly, CVD, epilepsy (Lower seizure threshold ), people with constipation, prostatic hypertrophy, raised IOP- as these will worsen with anti muscarinic affects
• Take care with hepatic insufficiency as sedative effects increase
• Used during pregnancy but can cause neonatal tachycardia and irritability
• No dosage change in renal insufficiency
• DO not use with MOAIS- wait 2-3 weeks. They both increase serotonin and noradrenaline, can increase hypertension hyperthermia, or serotonin syndrome
• Can augment, antimuscarinic, sedative, or hypotensive effects of other drugs (e.g alcohol, antihistamine, opioids)
• Take care with other drugs that prolong QT internals and also with stimulants of the SNS
• Do not give with anti malarial drugs lumefantrine and Artemisia derivatives

They take 2 weeks to start working, consider additional emergency treatment such as ECT if delay poses an unacceptable risk Depression: SSRI have less SE and less dangerous in overdose therefore before using this should switch to a different SSRI. Lofepramine seems to have less SE than others
Neuropathic pain:MUCH LOWER DOSES

  • Choose a sedative and less sedative drug depending on how agitated they are
  • Advice them to take their tablets last thing at night to minimize the impact of anticholinergic
  • Improvement in sleep is often the first benefit
  • Limit quantities and review before issuing a repeat Amoxapine, desipramine, and dothiepin are associated with the highest risk of death when taken in overdose
  • When time to stop, should reduce the dose slowly over 4 weeks,
  • Symptoms reviewed 1-2 weeks after starting and regularly there after
  • No effect seen at 4 weeks, consider changing dose or drug
  • Otherwise not adjusted until 6-8 weeks of therapy
  • Often do not respond to due to inadequate dose however elderly may be not respond to high dose
92
Q

What are Selective serotonin re-uptake inhibitors (SSRIs)

  • Fluoxetine
  • Sertraline
  • Paroxetine
  • Fluvoxamine
  • Citalopram
  • Escitalopram
A
  • First line in moderate to severe depression and mild if psychological therapies fail
  • Panic disorder and social phobias
  • Treatment of bulimia nervosa
  • Treatment of generalized anxiety and post traumatic stress
  • Obsessive compulsive disorder

Preferentially inhibit neuronal reuptake of 5HT from synaptic cleft. Therefore increasing its availability for neurotransmission. This is the mechanism by which they improve mood and physical symptoms in depression and help relieve the symptoms of panic and obsessive disorder. Different from tricyclic antidepressants, do not inhibit noradrenaline uptake and cause less blockade of other receptors, however SSRI are preferred as fewer adverse effect, less dangerous in overdose

Very common but mild and transient
• Gi upset, appetite and weigh disturbance (loss or gain)
• Headache
• Hypersensitivity reaction (including skin rash)
• CNS symptoms: agitation, anxiety, tremor, akathisia, nervousness, and insomnia
• Hyponatremia especially in elderly may present with confusion, reduced consciousness - due to syndrome of an appropriate ADH secretion, more common with TCA
• Suicidal thoughts and behavior may be increased especially when <25 years old
• Patients with increased suicide risk factors should be carefully evaluate
• Provide only 2 weeks supply to avoid overdose
• Lower seizure threshold
• Some (e.g citalopram) prolong QT interval- increase arrythmias
• Increase bleeding risk
• At high rose and overdose or in combination with other antidepressant classes, SSRI can cause serotonin syndrome- autonomic hyperactivity, altered mental status,, NM excitation
• Sudden withdrawal: cause GI upset, neurological and influenza like symptoms and sleep disturbance

caution prescribing when adverse effects risk:
• Epilepsy
• Peptic ulcer or bleeding disorders
• Young people- poor efficacy and associated with increased risk of self harm and suicidal thoughts so only by specialist
• SSRI metabolized by liver therefore reduced dose in liver impairment and also reduce in renal insufficiency
• NOT given with MOI as both increase synaptic serotonin levels together may lead to serotonin syndrome - wait 2-3 weeks before starting
• Gastroprotection with aspirin or NSAIDs due to bleeding risk
• Bleeding risk increased with anticoagulants
• Drugs that prolong QT interval such as antipsychotics
• Due to stimulant effect- contraindicated in bipolar and mania
• Although they have few anti cholinergic take care with prostatic enlargement or closed angle glaucoma
• Use not recommended with ECT
• Do not give with anti malarial drugs lumefantrine and Artemisia derivatives

NOTE that these are no more effective but they are better tolerated, they are less cardiotoxic in overdose
Better tolerated: citalopram and sertraline
They take 2 weeks to start working, consider additional emergency treatment such as ECT if delay poses an unacceptable risk
Rapid onset: avoid fluoxetine, half life longer, however may be advantage when compliance erratic
Paroxetine and fluvoxamine are slightly less well tolerated
When concomitant medications sertraline may be best

NOTE: ONLY FLUOXETINE IS BENEFIT FOR TREATING IN CHILDREN

  • Symptoms improve over a few weeks
  • Discuss psychological therapy
  • Keep going with 6 months
  • Warn not to stop suddenly
  • When do stop, reduce dose slowly over 4 weeks
  • Symptoms reviewed 1-2 weeks after starting and regularly there after, if no effect at 4 weeks consider changing dose or drug
  • Otherwise dose not adjusted until 6-8 weeks of therapy
93
Q

How does lactulose reduce hepatic encephalopathy?

A

For hepatic encephalopathy note it is also given for constipation

Inhibits the intestinal ammonia production by various mechanism
Conversion of lactic acid and acetic acid-> acidification of the stools therefore ammonia gets converted to ammonium this is less easily absorbed- and reduces the plasma ammonium.

Gut acidification inhibits ammoniagenic coliform bacteria leading to increased levels of nonammoniagenic lactobacilli.
Therefore reduced ammonia reduces the hepatic encephalopathy
Reduces the colonic bacterial load

Abdominal pain, diarrhoea, flatulence, nausea and vomiting

Galactosaemia, intestinal obstruction, perforation

94
Q

What are alpha blockers?

Non selective drugs:
Phentolamine
Phenoxybenzamine

A1 selective drugs:
Prazosin
Tamsulosin - only for BPH
Doxazosin - For hypertension and BPH 
Indoramin 
Terazosin 
Alfuzosin
A

Non selective a blockers are specialized:
• Treatment of hypertension due to pheochromocytoma
• Treatment of hypertension due to interactions between amines and MAOS
• Treatment of vasospasm (e.g due to ergotism)

Alpha 1 selective drugs are used for treatment of:
• 1st line for BPH when lifestyle changes are not sufficient, 5a reductase inhibitors may be added in selected cases. Surgical treatment may be an option especially if evidence of urinary tract damage
• Add on for hypertension when resistant

• Blocking actions of alpha receptor activation
• A1: arteries and veins
○ Blocks constriction in arteriolar resistance vessels- leading to relaxation and a fall in BP
• A1/A2:
○ Liver and pancreas: blocks glycogenolysis, gluconeogenesis, lipolysis
○ Uterus: blocks contraction
○ Bladder sphincter: blocks contraction therefore these is reduced resistance to bladder outflow
○ Pupil: blocks dilation

Most are highly selective for the a1 which are found mainly on smooth muscle- in the blood vessels and urinary tract (bladder neck and prostate) therefore stimulation contraction blockade and causes relaxation
Therefore leads to VASODILATION AND REDUCED RESISTANCE TO BLADDER OUTFLOW

SE:
• Postural hypotension , dizziness, syncope- especially after the first dose like with ACE
• Dry mouth
• Tachycardia
• Headache
• Can cause erectile impotence and piarism
• Indoramin can cause sedation therefore do not do skilled motor task
SE:
Avoid combining anti hypertensive drugs result in additive BP lowering effects but this may be the aim. Maybe avoid one of the doses e.g with B B which inhibit the reflex tachycardia that forms part of the compensatory response to vasodilation
• Not given in patients who already have hypotension (may omit 1 anti hypertensive on the day the alpha blocker is started)
• Avoid in patients with postural hypotension
• Do not give indoramin with MAOIS or linezolid can cause tyramine like reactions
• Alcohol enhances effect of indoramin

TAKE DRUG IN BED- FIRST DOSE
Start low then titrate up
Check efficacy in urine symptoms and/or BP
Safety for adverse effects by symptom enquire, measure standing and lying BP
Why reviewing of symptoms is key in patients hx:
• Although hypertension a blockers is usually used in patients who do not respond to other drug classes, many men with hypertension may also have BPH- therefore may introduce doxazosin at early stage
• Can improve both conditions with one drug

Note with pheochromocytoma: non selective are used in ST together with BB before and during surgery. Alpha blocker is used before BB- as giving Bb FIRST CAN CAUSE SEVERE HYPERTENSION

95
Q

What is co-amilofruse

A

Combination of amilonide (k+ sparing) and frusemide (loop)

hydrochlorothiazide (thiazide) as co-amilozide

96
Q

What is co-drydamol and co-codamol?

A

Opioids compound preparations
• Mild to moderate pain: as second line when simple analgesics such as paracetamol are insufficient.- second on the rung of pain ladder

• Paracetamol is poorly understood MOA- likely weak inhibitor of COX- increases the pain threshold

  • Codeine and dihydrocodeine are weak opioids they are metabolised by cytochrome P450 enzymes into morphine and morphine related metabolites which are agonists of opioids u receptors
  • Combing the two analgesics allows for better pain control than w/ either drug alone- putting together in fixed ratio improves convenience but cost of reduced flexibility in terms of dose titration
  • AE from paracetamol are rare
  • Common SE from codeine and dihydrocodeine: nausea, constipation, drowsiness
  • Overdose:
    • Hepatotoxicity for paracetamol
    • Opioid toxicity: neurological and respiratory depression
  • Caution with respiratory disease which is significant
  • Dose reduction in hepatic and renal impairment and elderly

Note with other sedating drugs e.g antipsychotics, benzodiazephrines, TCA but when unavoidable monitor them

All tablets contain 500mg of paracetamol but the dose of the opioid varies e.g co-codamol 8/500 or co-dydramol 10/500- first number ifs the amount of opioids in each tablet e.g 8mg codeine and 10 mg dihydrocodeine

When prescribe for regular- simulative laxative

Efficacy is about symptoms or pain score
Mild-moderate pain review 1-2hrs after oral dose chronic pain 1-2 weeks as see if step down

USE THE DRUGS AS SEPARATE WHEN PRECRIBING INTIALLY - as this way they can adjust the dose with your guidance to get the optimum balance between efficacy and SE w/o needing new prescription each time
Combination analgesics should be avoided as first-line treatment. Prescribing single constituent analgesics allows independent titration of each drug.

97
Q

Amiodarone what is it?

A
  1. Management of tachyarrhythmias including AF, atrial flutter, supraventricular tachycardia (SVT), ventricular tachycardia (VT) and refractory ventricular fibrillation (VF)

Used when other therapeutic options (Drugs or electrical cardioversion are ineffective or not appropriate

  • Various effects on myocardial ells- blocks Na+, Ca2, K+ channels
  • Antagonist to a and B adrenergic receptors
  • Reduce automaticity, slow conduction velocity, increase refractoriness, including in the AV node
  • By acting at the AVN- reduces ventricular rate in AF and atrial flutter
  • Through other effects, it may also increase the chance of conversion to and maintenance of sinus rhythm
  • SVT- re entry circuit includes the AVN and amidoamine may break and restore sinus rhythm
  • It suppresses spontaneous depolarisation- therefore treats/prevents VT
  • Little clinical trails but it does work in refractory VT

• Compared with other antiarrhythmic drugs causes little myocardial depression but can cause hypotension during IV infusion
• Chronic use:
• Pneumonitis
• Bradycardia and AV block
• Hepatitis
• Photosensitivity and grey discolouration on skin
Avoid sunlight and avoid grapefruit juice
• Thyroid problems = hypo/hyper
Due to being similar to thyroid hormone and containing iodine

LONG HALF LIFE therefore after elimination may take a long time to be eliminated

Only use when benefit> risk 
Avoid in:
	• Severe hypotension
	• Heart block 
	• Active thyroid disease 

Various drug interactions
Increases plasma concentrations of:
• Digoxin, diltiazem, and verapamil- may increase risk of bradycardia, AV block, heart failure (therefore half the doses)

Always requires senior involvement- do not do as foundation doctor (when continuing it via prescription always ask a consultant if in doubt as may have been a loading dose where need a maintenance dose)
EXCEPT IN CARDIAC ARREST - Where given for VF or pulses VT immediately after 3rd shock in ALS - dose is 300mg IV followed by 20mL of 0.9% sodium chloride or 5% glucose as flush - administered first then prescribed later. Here it is given as a bolus it is often in pre filled syringe, it should be given through best IV canula.

Outside of cardiac rest- if continuous or repeated IV fusions are anticipated- should be via central line- as peripheral IV can cause significant phlebitis (inflammation of vein)

Monitor: efficacy via HR and rhythm, safety by cardiac monitoring through baseline tests- renal, liver, thyroid and chest X ray

98
Q

What drug should be avoid in G6PD deficiency?

A

Quinine- used for malaria P. falciparum and leg cramps

99
Q

Quinine- Indications, MOA, SE, CI

A
  • Treatment/prevention of nighttime leg cramps only when they disrupt sleep regularly and when non pharmacological methods e.g passive stretching have failed
  • First line treatment of Plasmodium falciparum malaria

Leg cramps- due to sudden painful involuntary contraction of skeletal muscle. Thought to act by reducing the excitability of the motor end plate in response to ACH stimulation. This reduces the freq of muscle contraction.
In malaria- MOA is not understood but leads to rapid killing

Very toxic and fatal in overdose
	• Tinnitus 
	• Deadness
	• Blindness
	• GI upset 
	• Hypersensitivity 
	• Prolongs QT--> arrythmias 
	• Hypoglycemia which can be problematic as malaria patients are more likely to have it

Caution in patients with hearing or visual loss
Teratogenic therefore not in first trimester but in malaria benefit may outweigh the risk
G6PD deficiency (glucose-6 phosphate dehydrogenase) may precipitate hemolysis
Avoid with drugs that prolong QT e.g amiodarone, antipsychotics, quinolones, macrolides, and SSRI

Nocturnal leg cramps should be given at night for treatment of malaria higher doses may be needed
Prescribed orally in uncomplicated cases alongside another anti malaria e.g doxycline for IV in severe
IV doses depends on weight
IV quinine should be given as slow infusion
4 trial is given if no improvement then should stop
Ask to report an AE e.g hearing loss, visual disturbance, palpitations and it is harmful for ears, eyes, and heart
If they continue to take after 4 weeks- then review again at 3 months to see if not then consider discontinuing
Aim to avoid LT due to serious SE

Before starting check for reversible causes e.g electrolyte disturbances and drugs like statins, and B agonist and attempt non pharmacological e.g passive stretching

100
Q

What antieleptic drug is likely to lower INR in warfarin users?

A

Carbamazepine- ‘inducer’ of certain CYP enzymes; that is, it interacts with the regulatory regions of the genes to increase their transcription. The resulting increase in the amount of enzyme allows warfarin to be metabolised more rapidly. This means that less warfarin is available to inhibit clotting factor production, so its anticoagulant effect is diminished and the international normalised ratio (INR) falls. This puts the patient at risk of thromboembolic complications.

101
Q

What anti epileptic drug would increase INR in warfarin users?

A

Valproate is a CYP inhibitor, so its effect would be to increase the INR.

102
Q

What is the monitoring of low molecular weight heparin?

A

Antifactor Xa assay. Low molecular weight heparins (LMWHs), e.g. dalteparin, promote binding of antithrombin to factor Xa, inhibiting the final part of the clotting cascade to prevent production of a fibrin clot. LMWH-related anticoagulation can be quantified with an antifactor Xa assay. This is not required routinely, but it may be useful when the LMWH effect is less predictable, such as in pregnancy, renal impairment and significantly abnormal body weight. Platelet count and serum potassium should be measured in patients receiving LMWH for >4 days to monitor for side effects (heparin-induced thrombocytopenia, hyperkalaemia), but this is not an indicator of anticoagulation.

103
Q

What kind of drug is amiloride?

A

potassium-sparing diuretic

104
Q

What kind of drug is Chlortalidone?

A

Thiazide like diuretics

105
Q

What is a major SE of amlodipine and what can be used to treat it?

A

Ankle swelling is a common adverse effect of treatment with dihydropyridine calcium channel blockers. It is understood to be due to preferential dilatation of the pre-capillary arterioles, relative to post-capillary venules, causing increased pressure in the capillary bed and therefore greater filtration of fluid from vessels to the interstitium

Drugs with a venodilation effect, notably ACE inhibitors and angiotensin receptor antagonists, may help.

106
Q

What is the magic number in diabetes?

A

48 mol/mol. Haemoglobin A1c (HbA1c) is used in three main ways:

  1. To diagnose diabetes mellitus.
  2. To set a target level of glycaemic control with an existing therapeutic regimen (e.g. to guide dosage titration).
  3. To trigger intensification of treatment, by offering another agent.
107
Q

what are the 3 values that are key to remember

A
  • HbA1c values ≥48 mmol/mol can support a diagnosis of diabetes mellitus.
  • Patients treated by lifestyle measures should target HbA1c levels <48 mmol/mol, and drug treatment should be started if this is not achieved.
  • Patients taking a single agent should also target HbA1c <48 mmol/mol
  • those on two or more agents the target is <53 mmol/mol (recognising that treatment with two agents carries more risks and burdens, which may offset its glycaemic control benefits).
  • trigger for intensification of pharmacological treatment (i.e. adding another agent) is HbA1c ≥58 mmol/mol.
108
Q

Metformin MOA, indications, SE, CI?

A

T2DM As a first choice medication for control of blood glucose used alone or with other oral hypoglycaemic drugs (e.g sulphonylureas, DPP4 inhibitors) or insulin

HEPATIC GLUCOSE OUTPUT
• Lowers BG by increasing sensitivity to insulin
• Reduces glycogenolysis and gluconeogenesis- reduces hepatic glucose output
• Increases glucose uptake and use in skeletal muscle
• Decreases intestinal absorption of glucose
• Diverse intracellular mechanism
• Does not stimulate pancreatic insulin therefore no hypos
• Can cause weight loss, reduces insulin R, slows deterioration

Cellular mechanism complex- activates AMP kinase- cellular metabolic sensor- activation ahs diverse effects

GI effects e.g nausea, vomiting, taste, anorexia, diarrhoea
May cause weight loss

Lactic acidosis rare (not usually in stable but can occur in intercurrent illness, which ncrease metformin accumulation (e.g worse in renal) or with increase lactate production (e.gsepsis, hypoxia), or low lactic metabolism (e.g liver failure)

  • Renal failure and impairment (with impairment can reduce dose) <
  • Withheld in AKI E.g shock, sepsis, dehydration
  • Withheld in severe tissue hypoxia (sepsis,Cardio/resp failure MI)
  • Caution in hepatic impairment increase lactic acidosis
  • Withhold in acute alcohol intoxication precipitates lactic acidosis
  • Caution in chronic alcohol overuse (risk of hypos)
  • IV contrast (e.g for CT, coronary angiography) - may cause renal impairment, metformin accumulation and lactic acidosis WITHHELD BEFORE AND FOR 48HRS AFTER
  • ACE, NSAIDS, diuretics may cause renal impairment therefore renal function monitoring - caution use
  • Prednisolone, thiazide, loops increase glucose- oppose action and reduce efficacy

Given orally, give low dose then increase due to GI effects
Standard release +/- food

LT treatment- only stop or change if AE are intolerable or new contraindications
Give with food - if the GI effects are intolerable change to modified release
Excreted unchanged by kidney

Use with lifestyle, control BG low complications
Always tell if IV contract, check renal function
Always tell a doctor if having Xray or operation as may need to stop
Check blood glucose

Lactic acidosis symptoms: experience vomiting, stomach ache, muscle cramps, difficulty breathing severe tiredness- means that they may need to stop or withhold

ASSESS BLOOD GLUCOSE USING HBA1C: target with single agent <48mmol/mol
Identified if >58 then a new target is set of 53< mmol/mol
This balances the risk of hyperglycaemia against risk of treatment esp hypos
Measure renal function before and then annually
More often renal tests if at risk of impairment or deteriorating

Physical therapy should be tried for 3 months before starting treatment for Type 2 started if Hba1c >48mmol/mol
Insulin and drugs increasing insulin e.g sulphonylureas cause weight gain, can worsen DM in long term
But metformin does not therefore first line unless it is CI

109
Q

What kind of drug is glicazide?

A

Sulphonylureas lower blood glucose by stimulating pancreatic insulin secretion

110
Q

What are

Tamoxifen, anastrozole, letrozole

A

Sex hormone antagonists for breast cancer

  1. Early and locally advanced oestrogen (oestrogen) receptor positive ER positive breast cancer- adjuvant treatment option after surgery to reduce risk of recurrence
  2. Advanced ER positive breast cancer- to slow disease progression. Hormonal therapy is given first line if the tumour does not express human epidermal group factor receptor 2 positive (HER2 negative b/c other options are available with HER2 positive disease). Aromatase inhibitors are used in post menopausal women. Tamoxifen in those who cannot take aromatase inhibitors post menopausal and pre menopausal

2/3 of breast cancer are ER positive- oestrogen receptor positive. Oestrogen binds to these and stimulates cell proliferation
Sex hormone antagonists reduce tumour cell proliferation by blocking the effects of oestrogen.

Tamoxifen is a selective oestrogen receptor modulator (SERM) which acts to prevent oestrogen binding to receptor.

Aromatase inhibitors infer with synthesis of oestrogen outside of the ovary e.g in fat and muscle by inhibiting aromatase that converts androgens to oestrogen

Aromatase > tamoxifen in ER positive breast cancer post menopausal but not for pre menopausal- as they have little effect on ovarian oestrogen

111
Q

What are some SE/CI for Tamoxifen, anastrozole, letrozole (in particular for tamoxifen- antidepressants and anti coagulant)

A
Most common AE are symptoms of oestrogen depletion: 
	• Vaginal dryness
	• Hot flushes 
	• Loss of bone density 
Tamoxifen:
	• Venous thromboembolism
	• Endometrial cancer
Therefore report any unusual vaginal discharge or menstrual irregularity 

Other SE: Gi upset and headache
Rarely: agranulocytosis and liver failure

Tamoxifen: Increase risk of miscarriage therefore CI in pregnancy
Secreted in breast milk therefore avoided in lactation
Aromatase inhibitors not in per menopausal women unless their ovary function is suppressed or ablated (oophorectomy)

Tamoxifen inhibits cytochrome P450 responsible for metabolizing warfarin increasing risk of bleeding
SSRIs fluoxetine and paroxetine inhibits the hepatic activation of tamoxifen
Aromatase inhibitors have no important drug interactions

Taken orally
Tell them they reduce the risk of recurrence or slow the growth of the tumour, most takes 5 years
Warn them of SE of inhibited oestrogen

Warn tamoxifen of risk of endometrial cancer (report vaginal bleeding) and risk of blood clots- avoid in pregnancy

Check liver function and obtain FBC before starting tamoxifen and repeat if clinical suspicion
Combing warfarin and tamoxifen should be avoided due to drug interaction of increase risk of bleeding - LWH is superior to warfarin in cancer treatment patients

112
Q

What is the indication, MOA,SE, CI of sulphonylureas?

A

Sulphonylureas=Gliclazide

In type 2 diabetes
· Used with other drugs where blood glucose is not adequately controlled on single agent (usually metformin and/or other hypoglycemic agents)
· as a single agent to control blood glucose and reduce complications where metformin is contraindication or not tolerates

· Lowers blood glucose by stimulating pancreatic insulin secretion
· Block ATP dependent K+ channels in pancreatic B cells-depolarization of cell membrane
· Opening of voltage gated Ca2+ channels
· Intracellular Ca2+ -insulin secretion
· Only effective with residual pancreatic function
· Insulin=anabolic hormone therefore associated with weight gain=increases insulin resistance- worsen DM in long term

· Gastrointestinal upset (nausea, vomiting, diarrhea, constipation)- usually mild and infrequent
· Hypoglycemia is potentially severe adverse effect with high doses or where drug metabolism reduced, or where other hypoglycemic drugs are prescribed
• The hypoglycemia may last for hours and if severe should be in hospital
· Stimulates appetite and provokes weight gain due anabolic actions of insulin
· Rare hypersensitivity reactions include hepatic toxicity (e.g cholestatic jaundice), drug hypersensitivity syndrome (rash, fever, internal organ involvement), hematological abnormalities (e.g agranulocytosis)

· Reduce dose in hepatic impairment
· Blood glucose monitored in renal impairment
· Caution with increased risk of hypoglycemia including those with hepatic impairment (reduced gluconeogenesis), malnutrition, adrenal or pituitary insufficiency (Reduced counter regulator), and elderly
· Risk of hypoglycemia with other anti diabetics: metformin, DPP4 inhibitors, thiazolidinediones and insulin, and by alcohol
· Beta blockers may mask symptoms of hypoglycemia
· Reduced by drugs which increase blood glucose (e.g prednisolone, thiazide, and loop diuretics)
NOTE: Sulphonyl urea does not work for long, the sugars remain high

Several forms to choose from
Shorter duration and hepatic metabolism (e.g gliclazide) easier and particularly in elderly with impaired renal function
Increase gradually into BG controlled
Taken with meals
Explain about lifestyle measures
Warn them of signs with hypoglycaemia- e.g dizziness, nausea, sweating, confusion
Must take something sugar and starchy
Assess BG by HBA1C: treat <48
Add another when >58 new target is 53
Measure renal and hepatic function before treatment
Often this is given to elderly with renal impairment as first line
During acute illness, insulin R increases and renal and hepatic function may be impaired
Oral hypoglycaemias less effective at controlling BG and therefore SE occur e.g hypoglycaemia
Insulin treatment may therefore be needed ST in severe illness and has a short half life therefore dose adjusted more easily than oral

113
Q

What is pioglitazone?

A

Thiazolidinediones
· Treat T2DM
· Single agent: overweight where metformin is contraindicated or not tolerated
· Second agent: to metformin or sulphonylrea where blood glucose is inadequate on one and combination of metformin and sulphonylurea Is contraindicated or not tolerated
· Third agent: when metformin and a sulphoylurea where blood glucose is inadequate as an alternative to insulin

Insulin sensitizers, increases sensitivity to insulin in muscle, adipose and liver
Lower blood glucose via acting on PPARy receptors
Induces genes enhance insulin action in skeletal muscle adipose and liver
Increased glucose uptake peripherally and utilization and reduced hepatic gluconeogenesis
Do not stimulate insulin secretion do not cause hypoglycemia
Do cause weight gain, can increase insulin resistance

· GI upset, anemia, neurological effects
· Odema cardiac failure especially with insulin
· Small risk of bladder cancer and increase bone fractions
· Risk of heart failure and fluid retention
· Severe liver toxicity

· People at risk of serious adverse effects:
o Heart failure, cardiovascular disease
o If known bladder cancer or macroscopic hematuria or risk factors for bladder cancer
o Careful with elderly due to cardiac disease, bladder and bone fractions
o Hepatic impairment
o Used in combination with antidiabetic meds due to increase adverse effects of hypos and cardiac failure
· Avoid in pregnancy and breast feeding but can use in renal dysfunction

· With HbA1c and with liver enzymes before and during
· Should not replace life style
· Seek advice with unexpected side effects

114
Q

What of the diabetes drugs is associated with an increase risk of pancreatitis

A

Dipeptidylpeptidase-4 inhibitors-sitagliptin, linagliptin, saxagliptin

115
Q

sitagliptin, linagliptin, saxagliptin, MOA, SE, CI

A

Dipeptidylpeptidase-4 inhibitors
Type 2 diabetes:Incombinationwith metformin (and/or other hypoglycaemic agents) where blood glucose is not adequately controlled on a single agent. As asingle agentto control blood glucose and reduce complications wheremetforminis contraindicated or not tolerated.

  • Theincretins(glucagon-like peptide-1 [GLP-1] and glucose-dependent insulinotropic peptide [GIP]) are released by the intestine throughout the day, but particularly in response to food
  • They promote insulin secretion and suppress glucagon release, lowering blood glucose. The incretins are rapidly inactivated by hydrolysis by the enzymedipeptidylpeptidase-4(DPP-4). DPP-4inhibitors (‘gliptins’) therefore lower blood glucose by preventing incretin degradation and increasing plasma concentrations of their active forms
  • The actions of the incretins areglucose dependent, occurring when blood glucose is elevated, so they do not stimulate insulin secretion at normal blood glucose concentrations or suppress glucagon release in response to hypoglycaemia
  • This means that DPP-4inhibitors are less likely to cause hypoglycaemia thansulphonylureas, which stimulate insulin secretion irrespective of blood glucose.
  • Well tolerated
  • GI upset, nasopharyngitis, peripheral oedema
  • Hypos when prescribed in combination with other drugs e.g sulphoylureas or insulin
  • Small risk of acute pancreatitis
  • CI: in hypersensitivity, type 1 diabetes, ketoacidosis
  • Pregnancy, breastfeeding
  • Caution in elderly or history of pancreatitis
  • Reduce in moderate- severe renal impairment

Taken orally usually once daily
Also formulated with metformin

Advice to stop taking if they develop symptoms of pancreatitis e.g severe, persistent stomach pain to the back, allergy (Rash, swelling of the face, lips, tongue or thorat)

Assess blood glucose control by measuringHbA1c.When a DPP-4inhibitoris used as monotherapy, thetarget HbA1cis <48 mmol/mol; when used as part of combination therapy the target is <53 mmol/mol. An HbA1c>58 mmol/mol is generally atrigger to intensify therapywith another agent

116
Q

What drugs improve vascular complications of diabetes?

A

Metformin and a new class of antihyperglycaemic drugs, the sodium–glucose co-transporter 2 (SGLT-2) inhibitors (e.g. dapagliflozin, empagliflozin).

117
Q

What are dapagliflozin, empagliflozin

A

SGLT-2 inhibitors
Used in T2 diabetes

· Glucose is filtered freely in the renal glomeruli and reabsorbed in the proximal tubules
· SGLT2 is involved in reabsorption of glucose in the kidney similar
· Inhibition results in approximately 25% of the filtered glucose not being reabsorbed, with consequent glycosuria., similar to diuretics therefore reduces BP
· Improves cardiovascular outcomes
· Lower blood glucose and results in calorie loss and subsequent weight loss
SE:
• UTI and genital fungal infections
• Normoglycemia DKA

CI:
Renal impairment 
Type 1 diabetes 
Pregnancy 
Lactations
118
Q

What are Ipratropium, tiotropium, glycopyrronium, aclidinum- indications, MOA, SE, CI

A

Antimuscarinic (bronchodilators)
IUM

  1. COPD used to relieve breathlessness, short acting (ipratropium) e,g brought on by exercise and during exacerbation and long acting to prevent breathlessness and exacerbations
  2. Asthma short acting (used with short acting b2 agonist), long acting added with high dose ICS, and long acting B2 agonist (LABA) maintenance treatment in patients with one or more severe exacerbations in the year

Competitive inhibitor for ACH on muscarinic receptor stimulation leads to PSN effect rest and digest- increase HR and conduction, reduced smooth muscle tone (respiratory), reduced secretion from GI/resp, relax pupillary constrictor and ciliary muscles- pupil dilator- and inhibits accommodation

Reduced systemic absorption
Rapidly absorbed from lungs into circulation where they are inactivated hydrolysis
Irritation the respiratory tract- nasopharyngitis, sinusitis, cough
GI disturbance mouth may occur (chew gum, sweets, keep bottle of water) or constipation, urinary retention, blurred vision, headaches

Angle closure glaucoma- may increase intraocular pressure
Used in caution with arrythmias, urinary retention
But inhalation is less

Given as a inhaler
SAMA: ipratropium can be taken up to x4 daily or as needed when feel breathless
Higher dose nebulization during acteu attack
LAMA e.g iotrium, glucopyrronum regular x2 or x1 daily

Inhaled medication comes in various inhalers- choice of medicine is directed at the device that suits the patient
Nebulisation: liquid in chamber below the mask covering the mouth and nose- gas usually O2 bubbled through to vaporise the medicine to inhale
TYPE2 respiratory failure: medical air is used

NOTE: that short acting anti muscarinic can only be prescribed x4 a day any more increases AE but B2 agonists more frequently

119
Q

What is Leukotriene receptor antagonist montelukast, MOA, SE, CI

A

Montelukast, zafirlukast

  1. Adults:leukotrienereceptor antagonists may be considered as an add-on therapy forasthma, where symptoms are not adequately controlled by inhaledcorticosteroidsandlong actingβ2agonists (LABAs).
  2. Children aged 5–12 years:as an alternative toLABAsas an add-on therapy where inhaledcorticosteroidsare insufficient to controlasthmasymptoms.
  3. Children aged under 5 years:as a first-line preventative therapy in young children withasthmawho are unable to take an inhaledcorticosteroid.

• In asthma,leukotrienes produced by mast cells and eosinophils (amongst other sources) activate theG protein-coupledleukotrienereceptor CysLT1.This activates a cascade of pathways that result in theinflammation and bronchoconstrictionthat contribute to the pathophysiology of asthma.Leukotrienereceptor agonists reduce inflammation and bronchoconstriction in asthma by blocking the CysLT1 receptor and damping down the inflammatory cascade.

Well tolerated
Headaches
Abdominal pain

Increased risk of URTI
Uncommonly hyperactivity and reduced ability to concentrate

Churg strauss syndrome has been associated where eosinophilic autoimmune disorder
Arthralgia, bruising, GI upset, infection and headaches

Only used when incomplete control with inhaled ICS, and long acting B2 agonist

Oral

The place in asthma therapy forleukotrienereceptor antagonists continues to evolve. It may be that some subgroups of patients are more likely to benefit (e.g. those with exercise-induced bronchoconstriction or rhinitis-associated asthma), and some are less likely to benefit (e.g. children with viral-induced wheeze).

120
Q

What kind of steroid is given in Addison’s crisis?

A

Hydrocortisone as it is the only one that can be given IV

121
Q

What antibiotics are often used for H. pylori (in combination with PPI)- note that more than one is always used due to risk of resistance

A

The various regimens considered acceptable for H. pylori eradication, including recommended drug doses, are set out in a helpful table in the BNF. Options for proton pump inhibition include lansoprazole, omeprazole and pantoprazole. The antibiotics are selected from amoxicillin (a broad-spectrum penicillin), clarithromycin (a macrolide) and metronidazole.

122
Q

What are Prochlorperazine, chlorpromazine

A

Anti emetics -Phenothiazines

Prophylaxis and treatment of N/V in a wide range of conditions when due particularly to vertigo - but due to SE other classes usually preferred

Psychotic disorders e.g schizophrenia when used as first generation typical antipsychotics

N/v triggered from various factors e.g gut irritation, drugs, motions, vestibular disorders as well as higher stimuli (e.g sights, smells, emotions) all converge on the vomiting centre in the medulla that gets input from the chemoreceptive trigger zone (solitary tract nucleus, which is innervated by the vagus, the vestibular system, and higher neurological centres

Via blocking various receptors including Dopamine D2 in the gut and CTZ, and to lesser extend, H1 and muscarinic receptors in the vomiting and vestibular system

This makes them effective for nausea/vomiting in various situations including chemo, radiotherapy, and vertigo

  • Drowsiness and postural hypotension
  • Extrapyramidal syndromes are major drawback - in ST likely as acute dystonic reaction e.g oculogyric crisis
  • In longer term use e.g antipsychotic use more likely to get tardive dyskinesia
  • Like all antipsychotics- QT prolongation
  • Sedative and potential hepatotoxicity- avoid in severe liver disease
  • Avoid in patients susceptible to anticholinergic SE e.g prostatic hypertrophy
  • Dose reduce in elderly
  • Consult BNF as various interactions
  • Drugs that prolong QT interval e.g antipsychotics, amiodarone, ciprofloxacin, macrolides, quinine and SSRIs

Get expert advice as other drugs should be tried first
IM should be given deep injection into large muscle

Not driving and postural hypotension

Prolonged use- monitor for extra-pyramidal features may be subtle

Note: that phenothiazines are rarely used as first line antiemetics in hospital- however haloperidol (first generation antipsychotics) quite common especially in opioid induced nausea- low dose does not get much SE

123
Q

What are

Ondansetron, granisetron

A

Anti emetics
Serotonin 5HT3 receptor antagonists

Prophylaxis and treatment of N/V in a wide range of conditions when due particularly due to general anaesthesia and chemotherapy

N/v triggered from various factors e.g gut irritation, drugs, motions, vestibular disorders as well as higher stimuli (e.g sights, smells, emotions) all converge on the vomiting centre in the medulla that gets input from the chemoreceptive trigger zone (solitary tract nucleus, which is innervated by the vagus, the vestibular system, and higher neurological centres

Serotonin plays important role in these pathways in two ways
1. High density of 5-HT3 receptors in the CTZ- which are responsible for sensing emetogenic substances in the blood (e.g drugs
2. Serotonin is the key NT in the gut which is released with emetogenic stimuli- acting on 5HT3 receptors stimulates the vagus nerve which activates the vomiting centre via solitary tract nucleus
Therefore effective with nausea and vomiting due to CTZ stimulation e.g drugs and visceral stimuli (gut infection and radiotherapy)

• Rarely • Constipation, diarrhoea, headaches

  • Small risk of prolonging QT but only really at high dose therefore avoid prolong QT interval- review an ECG before
  • Avoid drugs that prolong QT interval e.g antipsychotics, amiodarone, ciprofloxacin, macrolides, quinine and SSRIs- if in doubt check the BNF
  • Dose differs for each indication- high dose for chemotherapy induced nausea, and vomiting
  • Oral, rectal and injectable are available
  • Route of administration, whether give regular or as required- clinical indication
  • IM are painful therefore IV route preferable if oral/rectal are not appropriate
  • Oral drugs should eb given an hour before symptoms are anticipated while IV immediately before treatment or procedure

Recent review found no evidence of adverse fetal outcomes for ondansetron for severe morning sickness where benefits> risks
Morning sickness difficult to treat due to spontaneous abortion risk and fetal abnormalities

124
Q

What anti emetic is used for

  • general anaesthetisia/chemotherapy?
  • Vertigo (but high SE)
  • motion sickness or vertigo
  • Reduced gut motility
A
  • general anaesthetisia/chemotherapy- serotonin 5H3 receptor antagonists
  • Vertigo (but high SE)- phenothiazines
  • motion sickness or vertigo -histamine H1 receptor antagonists
  • Reduced gut motility-dopamine D2 receptor antagonists
125
Q

What is loperamide, indications, SE, CI?

A

Loperamide

ALSO KNOWN AS IMMODIUM

  1. Symptomatic treatment of acute diarrhoea
  2. Chronic diarrhoea
  3. Faecal incontinence
  4. Pain of bowel colic in palliative care

Prolongs the transit time of intestinal contents, reduces the daily faecal volume, increases viscosity and bulk density, diminishes fluid loss and electrolytes. It is an opioid receptor agonist and acts on the mu opioid receptors in the myenteric plexus large intestine, does not affect CNS. Works by decreasing the activity of the myenteric plexus, which decreases the motility of the circular and longitudinal SM of the intestinal wall. Increases the amount of time they stay in the intestine- allowing more water to be absorbed into- also decreases colonic mass movements and suppresses gastrocolic reflex

Gastrointestinal disorders;headache;nausea

Active ulcerative colitis;antibiotic-associated colitis;bacterial enterocolitis;conditions where abdominal distension develops;conditions where inhibition of peristalsis should be avoided
Caution in children <12

126
Q

What are fentanyl, methadone, hydromorphone, pethidine and oxycodone ?

A

strong opiates

127
Q

What are Cetirizine, loratadine, fexofenadine, chlorphenamine

A

Anti histamines- H1 receptor antagonists

  1. FIRST LINE allergies, especially hay fever
  2. Aid relief of itchiness (pruritus) and hivers (urticaria) due to insect bites, infections drug allergies
  3. ADJUCT in anaphylaxis, after adrenaline and other life saving measures
  4. Other drugs in this class may be used for N/V

· Antagonist of H1 receptor
· Histamine is released from storage granules in mast cell due to antigen binding to IgE on surface
· Via H1 receptors mainly, induces features of T1 hypersensitivity: increased capillary permeability (wheal), vasodilation causes erythema (flare), itch due to sensory nerve stimulation
· In the nasopharynx as in hay fever, causing nasal irritation, sneezing, rhinorrhoea, congestion, conjunctivitis, itch
· In skin, urticaria
· Widespread histamine release (anaphylaxis)- generalized vasodilation, vascular leakage- with hypotension
· Via blocking H1 receptors- blocks excessive histamine
· In anaphylaxis effect is too slow therefore adrenaline is first line

  • Sedation in first generation antihistamine (e.g chlorphenamine) as via H1 receptors histamine has role in brain wakefulness
  • Newer second generation (loratadine, cetirizine, fexofenadine) do not cross BBB so tend not to have effect
  • Safe in most patients
  • Sedating antihistamines (e.g chlorphenamine) should be avoided in severe liver disease as may precipitate hepatic encephalopathy - note in the TOP 100 QUESTION PRESCRIBED LORATADINE
  • Cetirizine (10mg tablets), loratadine (10mg tablets) and chlorphenamine (4mg tables and 2mg/5mL oral solution) may be purchased w/o prescription
  • Cetirizine and loratadine taken once daily
  • Chlorphenamine taken 4-6 hours
  • Anaphylaxis: chlorphenamine 10mg IV/IM but not prioritized over adrenaline and other life saving measures
  • May tell to take in evening with chlorphenamine - avoid driving or carry out activity that need concentration- avoid combining with alcohol
128
Q

What drugs should you be careful with with azathioprine

A

Not prescribed with xathine oxidase inhibitors such as allopurinol as they reduce its metabolism

Increased risk of leukopenia with myelosuppressive or drugs affecting purine synthesis like trimethoprim
Also may reduce the effect of warfarin

129
Q

What to avoid in severe and moderate pencillin allergy

A

When have a penicillin allergy -
Severe (e.g. anaphylaxis, bronchospasm, urticaria, angioedema)avoid all penicillin’s, cephalosporins
Non severe penicillin (mild rash) use cephalosporins and carbapenems with caution

130
Q

What antibiotic should you prescribe for a patient with a sore throat due to unknown origin

A

phenoxymethylpenicillin, notamoxicillin

If the sore throat is due to Epstein–Barr virus (glandular fever), amoxicillin treatment commonly causes a rash. Although this is not truly an allergic reaction, it may lead erroneously to a lifetime label of ‘penicillin allergy’ for that patient.

131
Q

Penicillin

Benzylpenicillin, phenoxymethylpenicillin are not active against which kind of bugs

A

Gram-ve bacilli rods

132
Q

What does Penicillin

Benzylpenicillin, phenoxymethylpenicillin reduce renal excretion of?

A

Methotrexate

133
Q

What are the indications, MOA, SE,CI of What does Penicillin
Benzylpenicillin, phenoxymethylpenicillin

A
  1. Streptococcal infection, including tonsillitis, pneumonia (with a macrolide or tetracycline), endocarditis (usually with gentamicin) and soft tissue and skin infections (with flucloxaccilin)
  2. Meningococcal infection e.g meningitis
  3. Clostridial infection e.g gas gangrene

Benzylpenicillin, phenoxymethylpenicillin have relatively narrow spectrum

Against some gram +ve (e.g streptococci, bacillus and some anaerobes e.g clostrida) and gram -ve (e.g Neisseria meningitidis, n. gonorrhoea

THEY ARE NOT ACTIVE VS. GRAM-VE BACCILI RODS

Penicillins inhibit the enzymes responsible for cross-linking peptidoglycans in bacterial cell walls. Thisweakens cell walls,preventing them from maintaining an osmotic gradient. Uncontrolled entry of water into bacteria causescell swelling, lysis and death.

Penicillins contain aβ-lactam ring,which is responsible for theirbactericidalactivity.

Side chainsattached to theβ-lactam ring can be modified to make semi-synthetic penicillins. The nature of the side chain determines the antimicrobial spectrum and other properties of the drug.

Bacteriaresistthe actions of penicillins by makingβ-lactamases,enzymes which break theβ-lactam ring. Other resistance mechanisms include limiting the intracellular concentration of the drug (by reduced bacterial permeability or increased extrusion) and changes in the target enzyme to prevent penicillin binding.

As the antimicrobial spectrum of benzylpenicillin and phenoxymethylpenicillin is relatively narrow, they are often combined with other antibiotics (see Common indications).

Allergy effects 1-10% of people exposed usually skin rash 7-10days after first exposure or 1-2 days after repeat (Subacute delayed IGG mediated)

Less commonly 0.05% have immediate life threatening IGE mediated anaphylactic reaction with some or all of: hypotension, bronchospasms, orofacial, pharyngeal, laryngeal odema

Neurological toxicity (convulsions, coma) with high dose therapy or accumulation with severe renal impairment- therefore reduce dose

CI: penicillin allergy
Note that even to one type implies allergy to all due to the basic structure
Immediate hypersensitivity react to cephalosporins and other B lactam

All reduce renal excretion of methotrexate therefore increase risk of toxicity

134
Q

What is the deal with Pencillin antipseudomonal- piperacillin with tazobactam (e.g. Tazocin®) why do you prescribe them with tazobactam- what

A

Pencillin antipseudomonal
piperacillin with tazobactam (e.g. Tazocin®)

Antipseudomonal penicillins are reserved forsevere infections,particularly where there is abroad spectrum of potential pathogens;antibiotic resistanceis likely (e.g. hospital-acquired infection); or patients areimmunocompromised(e.g. neutropenia). Clinical infections treated with these drugs include:

1. Lower respiratory tract infections
2. Urinary tract infections
3. Intraabdominal sepsis
4. Skin and soft tissue infections.

Antipseudomonal penicillins (e.g. piperacillin) have abroad spectrumof activity against a wide range of Gram-positive and Gram-negative bacteria (notably includingPseudomonasspp.) and anaerobes. They are formulated with aβ-lactamase inhibitor (e.g. tazobactam), which confers antimicrobialactivity againstβ-lactamase-producing bacteria(e.g.Staphylococcus aureus, Gram-negative anaerobes).

Penicillins inhibit the enzymes responsible for cross-linking peptidoglycans in bacterial cell walls. Thisweakens cell walls,preventing them from maintaining an osmotic gradient. Uncontrolled entry of water into bacteria causescell swelling, lysis and death.Penicillins contain aβ-lactam ring,which is responsible for theirbactericidalactivity.Side chainsattached to theβ-lactam ring can be modified to make semi-synthetic penicillins. For piperacillin, the side chain ofbroad-spectrum penicillinshas been converted to a form of urea. This longer side chain may improve affinity to penicillin- binding proteins, increasing the spectrum of antimicrobial activity

GI SE
Antibiotic associated colitis
Delayed or immediate hypersensitivity

Anti pseudomonas used in caution with Patietns with increased risk of C.difficle - elderly and in hospital
CI: is pencillin allergy
Reduce dose in moderate- several renal

All penicillin’s reduce the renal excretion of methotrexate and also increase the anticoagulant effect of warfarin by killing normal GI flora

Piperacillin is always given with tazobactam. It can be prescribed using the drugs’ generic names (piperacillin–tazobactam) or by brand name (e.g. Tazocin®, although note that non-proprietary compound preparations are now available). Piperacillin–tazobactam can only be given by IV infusion. Thus if it is the only effective agent, the whole course (usually 5–14 days) must be given IV as no oral switch is possible. The usual dose is 4.5 g, containing 4 g of piperacillin and 500 mg of tazobactam, given every 6–8 hours
Each dose of piperacillin–tazobactam contains about 11 mmol Na+and is infused in 50–150 mL fluid (which may contain more sodium). Take this into account when determining the need for supplementary fluid and electrolyte therapy, particularly in patients with heart failure.

135
Q

Whats the deal with penicillin- penicillinase resistance floxacillin?

A

Staphylococcal infection usually part of combination therapy including:

1. Skin and soft tissue infection e.g cellulitis 
2. Osteomyelitis and septic arthritis 
3. Other infections like endocarditis

Penicillinase-resistant penicillins have anarrow spectrumof activity against Gram-positive staphylococci.

Penicillins inhibit the enzymes responsible for cross-linking peptidoglycans in bacterial cell walls. Thisweakens cell walls,preventing them from maintaining an osmotic gradient. Uncontrolled entry of water into bacteria causescell swelling, lysisanddeath.Penicillins contain aβ-lactam ring,which is responsible for theirbactericidalactivity.Side chainsattached to theβ-lactam ring -flucloxacillin, an acyl side chain protects theβ-lactam ring fromβ-lactamases, which are enzymes made by bacteria to deactivate penicillin. Makes effective vsβ-lactamase-producing staphylococci.Meticillin-resistantStaphylococcus aureus(MRSA) resists the actions offlucloxacillinby reducing penicillin-binding affinity. As penicillinase-resistant penicillins are active against staphylococci only, they are often combined with other antibiotics, particularly where the infecting organism has not been identified or infection is severe, e.g. withbenzylpenicillinfor severe cellulitis.

GI SE
Skin rash and anaphylactic reaction can occur

Liver toxicity including cholestasis and hepatitis is rare but serious effect may occur up to 2 months after

Dose reduction in renal failure
CI: hx of penicillin allergy
CI in prior flucloxacillin related hepatotoxicity and should be used in caution with hepatic impairment
Reduces renal secretion of methotrexate- increased toxicity

IV for severe infection and may be needed for deep seated infection e,g osteomyelitis and endocarditis for 6 weeks

Less severe can be given lower dose and short plasma life means every 6hrs

Note that a SE is vomiting this is not an allergy to penicillin

136
Q

Whats the deal with broad spectrum antibiotics? Amoxicillin, co-amoxiclav

A

Broad spectrum penicillin’s
Amoxicillin, co-amoxiclav`

  1. Amoxicillin used to treat a range of susceptible infections including uncomplicated CAP, otitis media, sinusitis, UTIS
  2. Treatment of H.pylori associated peptic ulcers- amoxicillin with clarithromycin, metronidazole and PPI
  3. Co-amoxiclav is common for severe, resistant, Hospital acquired infections including RT, GU, GI, cellulitis and bone and joint infections
    Amoxicillin is not generally used as risk of bacterial resistance in this setting

Broad spectrum wide range of gram +Ve and gram-ve cocci and bacilli
However inactivated by bacterial penicillinases and resistance is increasing prevalent
Addition of B lactamase inhibitor clavulanic acid restores activity against many resistant strains

Penicillins inhibit the enzymes responsible for cross-linking peptidoglycans in bacterial cell walls. Thisweakens cell walls,preventing them from maintaining an osmotic gradient. Uncontrolled entry of water into bacteria causescell swelling, lysisanddeath.Penicillins contain aβ-lactam ring,which is responsible for theirbactericidalactivity. Broad-spectrum penicillins are synthesised by addition of an amino group to theβ-lactam ring side chains, broadening activity against aerobic Gram-negative bacteria

GI SE
AB associated collitis
Pencillin allergy, anaphylaxis

Acute liver injury (Cholestatic jaundice or hepatitis) after or during co-amoxiclav generally short lived and stop once treatment stopped

CI: history of allergy
Caution at risk of C.difficle
Caution with history of penicillin associated liver injury
Dose reduce in severe renal injury due to risk of crystalluria
Broad spectrum enhance the anti coag effect of warfarin

Severe infection- amoxicillin at high dose for IV can be switched to oral after 48hrs if the clinically indicated and the patient is improving and able to take- this reduces complications, costs and early discharge

Mild-moderate without systemic features- oral should be given at lower dose
Note that oral suspensions of co-amoxiclav may stain teeth superficially

Always follow local guidelines broad spectrum increase risk of bacterial resistance and antibiotic associated infections therefore use narrowest until results back!

137
Q

What drugs should be avoided to take with Gavison?

A

Gaviscon® is a compound alginate which also contains the antacid calcium carbonate. The divalent cation (Ca2+) in calcium carbonate can bind many drugs in the gut and reduce their absorption. Examples include tetracyclines, digoxin, iron, bisphosphonates and thyroid hormones such as levothyroxine. In order to minimise this interaction, a 2-hour gap is advised between taking Gaviscon® and other medicines.

138
Q

Treatments of UTI’s

A

First-line options for an uncomplicated urinary tract infection (UTI) include trimethoprim, nitrofurantoin and amoxicillin (a broad-spectrum penicillin).

Trimethoprim, you should make sure that the patient is not pregnant. Trimethoprim is potentially teratogenic in the first trimester.

Nitrofurantoin should be avoided in the latter stages of pregnancy.

139
Q

Rifampicin- MOA, SE,CI

A

ANTI TB
2 months intensive
4 months continuation

• Inibits DNAA dependent RNA polymerase in bacterial cells

  • Red orange metabolites - within urine and eyes- will stain contacts
  • P450 inducer
  • Hepatitis - abnormal live function
  • Rash
  • GI upset
  • OCP and prednisoline decrease

• Care with warfarin, calcineurin inhibitor, oestrogen, phenytoin

CHECK LIVER FUNCTION

140
Q

Isoniazide- MOA, SE, CI

A

2 months intensive
4 months continuation

• Inhibits synthesis of long chain mycolic acid in cell wall

  • Rash
  • Hepatitis
  • Peripheral neuropathy (give B6= pyridoxine)- increase with DM, CKD, HIV, malnutrition

Increase with DM, CKD, HIV, malnutrition- peripheral neuropathy

CHECK LIVER FUNCITON

141
Q

Pyrazinamide- MOA, SE, CI

A

2 months intensive

Converts pyrazinamide into pyrazinoic acid which inhibits fatty acid synthase

  • Hepatitis- rare but serious liver toxicity
  • Facial flushing
  • Rash
  • Nausea
  • Anorexia
  • Hyperuricaemia gout
  • Arthralgia, myalgia

Reduce dose if ECF<30

CHECK LIVER FUNCITON

142
Q

Ethambutol- MOA, SE, CI

A

2 months intensive

  • Enzyme arabinosyl transferase which polymerizes arabinose into arabinan
  • May act as bacterial metal ion chelating agent
  • Dose related optic neuropathy -presents as red/green colour blindness- low visual acuity
  • Reduced acuity
  • Optic neuritis

Monitor levels if EGF <30

Check visual acuity -at start monitor for symptoms, monthly checks if treatment >2 months

143
Q

Bactericidal vs bacteriostatic antibiotics some examples of each

A

Bactericidal antibiotics kill bacteria, whereas bacteriostatic antibiotics prevent bacterial growth.

Bactericidal antibiotics include those that disrupt bacterial cell walls (e.g. all penicillins, cephalosporins, carbapenems, vancomycin) or inhibit DNA synthesis (e.g. quinolones, metronidazole, nitrofurantoin)

Bacteriostatic antibiotics generally are those that inhibit protein synthesis (e.g. macrolides, tetracyclines, trimethoprim). Chloramphenicol is generally considered to be bacteriostatic, but in high concentrations and with highly susceptible organisms it can be bactericidal.

In theory, bactericidal antibiotics should be more effective than bacteriostatic antibiotics. In practice, there is no clinically significant difference in efficacy. Bactericidal antibiotics may be considered necessary in deep-seated infections where antibiotic penetrance is poor (e.g. vegetations in endocarditis, osteomyelitis) or in situations where there is immunocompromise (e.g. neutropenia) or need for infection to be eradicated as rapidly as possible (e.g. meningitis)

144
Q

What is a good best guess treatment for community acquired pnuemonia

A

CAP: including Streptococcus pneumoniae (Gram positive), Haemophilus influenzae (Gram negative) and ‘atypical’ organisms such as Mycoplasma pneumoniae and Legionella pneumophila.

Doxycycline (a tetracycline) is suitable because it covers Gram-positive, Gram-negative and atypical organisms.

145
Q

What is unusual about the quinolone ciprofloxacin-

A

unusual as has significant activity against pseudomonas aeruginosa

146
Q

What drugs increase QT interval?

A

There are several causes of a prolonged QT interval. Drug causes include antiarrhythmics (e.g. amiodarone), antipsychotics (e.g. haloperidol), macrolide antibiotics (e.g. clarithromycin) and quinine

Combining drugs with QT-prolonging effects can be dangerous (clarithromycin with quinine sulfate for this patient) and should be avoided.

147
Q

At 48 hours what are the 5 principals of antibiotic treatment?

A

Stop antibiotics if there is no evidence of infection.
Switch antibiotics from IV to oral administration.
Change antibiotics – ideally to a narrower spectrum if improving (or broader if no improvement or deterioration).
Continue and review again at 72 hours.
Set up outpatient parenteral antimicrobial therapy where a longer course of IV treatment is required.

148
Q

What is used in treatment of methotrexate toxicity?

A

Folinic acid
Methotrexate inhibits the enzyme dihydrofolate reductase, which converts dietary folic acid to tetrahydrofolate (FH4). FH4 is required for DNA and protein synthesis. Folinic acid is readily converted to FH4 (without the need for dihydrofolate reductase) and is therefore useful in methotrexate toxicity.

149
Q

What is the deal with Alginates and antiacids

A

Gavison, peptac

  1. Treatment of GORD for symptomatic relief of heartburn
  2. Dyspepsia for short term relief of indigestion

Most often taken as compound preparations an alginate with one or more antacids e.g sodium bicarbonate, calcium carbonate, magnesium or aluminum salts

Work by buffering acids
Alginates increase the viscosity of stomach contents- reduces reflex
After reacting with acids they form a floating raft that separates the gastric contents from the gastro-esophageal junction to prevent the mucosal damage
There is some evidence that they also inhibit pepsin production
Antacids alone (usually aluminium or magnesium compounds) can be used for the short-term relief of dyspepsia

Few SE which vary depending on their constituents and the dose
Magnesium salts- diarrhea
Aluminum salts- constipation

Well tolerated and safe

Compound alginates should not be given in combination with ▴thickened milk preparationsas they can lead to excessively thick stomach contents that cause bloating and abdominal discomfort.
Salt and K+ should caution with fluid overload and hyperkaliemia (e.g renal failure)
Some preparations worsen Hyperglycaemia e.g with sucrose

Divalent cations in compound alginates bind to other drugs - they can reduce the [] of many drugs e.gThis applies toACE inhibitors, some antibiotics (e.g.cephalosporins,ciprofloxacinandtetracyclines),bisphosphonates,digoxin,levothyroxineandproton pump inhibitors (PPIs). By increasing the alkalinity of urine, antacids can increase the excretion ofaspirinand lithium.

Taken after meals, before bed and/or when symptoms
Temporary measure therefore discuss lifestyle
Advise to leave a gap of 2 hours

If there are persistent symptoms or ‘red flags’, such as bleeding, vomiting, dysphagia and weight loss, further investigation and specialist review are required

150
Q

Whats the deal with methotrexate?

A
  1. As part of DMARB for RA
  2. Chemotherapy regimen for cancers including leukaemia, lymphoma, and some solid tumours

Inhibits dihydrofolate reductase which converts dietary folic acid to tetrahydrofolate (FH4) which is needed for DNA/protein synthesis so lack of this prevents cellular replication- particularly sensitive are actively dividing cells

Anti-inflammatory and immunosuppressive effects
Mediated by inflammatory mediators e.g IL6, IL8, TNFaa although mechanisms not undertook

Mucosal damage (e.g sore mouth, GI upset)
Bone marrow suppression (neutropenia and increased risk of infections)
Hypersensitivity e..g cutaneous, hepatitis, pneumonitis

LT: hepatic cirrhosis, pulmonary fibrosis

Accidental overdose risk: renal impairment and hepatoxicity and neurological effects: headaches, seizures, coma

Pregnancy- use contraception during and 3 months after
Renally excreted- therefore CI in severe renal impairment
Avoid in abnormal live

Toxicity more likely with NSAIDS, penicillin’s, co prescription with other folate antagonists: trimethoprim, phenytoin increase risk of haematological problems
Risk of neutropenia highest with methotrexate and clozapine

Management of overdose: folinic acid which rescues the normal cells from methotrexate and hydration and urinary alkalinisation to allow excretion

Only given by specialists
Orally for autoimmune
Cancer: IV, IM or intrathecal

Warn them to get urgent medical advice if develop sore throat, fever ((infection), bruising or bleeding (low platelet count), nausea, abdominal pain, dark urine (liver poisoning) or breathless (lung toxicity)

Get methotrexate treatment booklet and warning card

Measure FBC, liver, renal function before then 1-2 weeks after until treatment is established then 2-3 times monthly after

151
Q

Whats the deal with chloramphenicol?

A
  1. Bacterial conjunctivitis using eye drops or ointment
  2. Otitis externa using ear drops

Due to toxicity- systemic oral or IV rarely used
In UK: used to life threatening infection e.g epiglottis (haemophilus influenzae) and typhoid fever (Salmonella spp)

Broad activity- gram positive, negative, aerobic, anaerobic
Binds to bacterial ribosomes, inhibiting protein synthesis
Therefore bacteriostatic (stops bacterial growth)- helps immune system clear
High doses- bactericidal

Most common bacterial resistance mechanism is production of acetyltransferase enzymes that inactivate the drug.
Other mechanisms include target modification, decreased membrane permeability, increased expression of efflux pumps. However most remain sensitive to due limited use

Topical: Minor: transient irritation, burning, stinging and sensitivity reactions: itching and dermatitis

Systemic administration: dose related bone marrow suppression more likely with high dose therapy or when drug accumulates due to impaired hepatic metabolism- occurs during treatment, reversed after withdrawal

Aplastic anaemia is idiosyncratic and has uncreditable relationship with dose may be delayed

Grey baby syndrome is circulatory collapse in exposed neonates- not able to metabolise

Optic and peripheral neuritis with prolonged systemic

Hypersensitivity
Personal or family history of bone marrow disorders

Systemic: CI in third trimester of pregnancy, breastfeeding and children <2 due to grey baby - topical avoided unless needed

Reduce and monitor with hepatic impairment

Topical administration: ear, eye, drops or ointment for eye
Eye ointment stays in for longer than eye drops
Otitis externa higher dose as less risk of systemic absorption
Never prescribe chloramphenicol without expert

Monitor systemic with specialist: Full blood count

152
Q

What is grey baby syndrome associated with?

A

Chloramphenicol